You are on page 1of 94

All India Open Mock Test - 1

Q1. The Constitution of India is called a ‘living document’ for which one of the following reasons?

a) It can accommodate the changing needs of society.

b) Its basic structure cannot be altered or amended even by legislation.

c) It reflects the dreams and aspirations of the founding fathers.

d) It adheres to the principle of natural justice duly recognised in Part III and IV.

Answer: a

Explanation:

• The Constitution of India accepts the necessity of modifications according to changing needs
of the society. Secondly, in the actual working of the Constitution, there has been enough flexibility
of interpretations. Both political practice and judicial rulings have shown maturity and flexibility in
implementing the Constitution. These factors have made our Constitution a living document rather
than a closed and static rulebook.

• The fact that our Constitution is a balance between rigidity and flexibility which enables us
to sustain the core basic structure while amending the rest of the structure according to the changes
in the society is proof that our Constitution is not only a living document but also a means to steer
towards the path of advancement.

• The provisions related to Right To Education, Amending Article 370, Goods and Service Tax,
National Commission of Backward Classes or the judgments related to the right to privacy, section
377, triple talaq or the NOTA judgment, show that just like Darwin pointed out that ‘it is not the
strongest of the species that survive, nor the most intelligent, but the one most responsive to
change’, similarly only with change and adaptation can our democracy survive.

• The Constitution is not merely a lengthy legal document but a charter of values and
principles; a dream of a free, just, and equal society.

Therefore, option (a) is the correct answer.

Subject: Polity | Constitution

Tags: Factual

Q2. Which of the following provisions of the Constitution of India have a bearing on Health?

1. Tenth Schedule

2. Eleventh Schedule

3. Directive Principles of State Policy

4. Fundamental Duties

5. Fundamental Rights

Select the correct answer using the code given below.


a) 1, 2 and 5 only

b) 3 and 4 only

c) 2, 3 and 5 only

d) 1, 2, 3, 4 and 5

Answer: c

Explanation:

• Tenth Schedule: This has provisions relating to disqualification of the members of Parliament
and State Legislatures on the ground of defection. This schedule was added by the 52nd Amendment
Act of 1985, also known as Anti-defection Law. So, point 1 is not correct.

• Eleventh Schedule: There are 29 subjects covered under this schedule, these include:
1.Cultural activities; 2. Markets and fairs; 3. Health and sanitation, including hospitals, primary
health centres and dispensaries; 4. Family welfare; 5. Women and child development; 6. Social
welfare, including welfare of the handicapped and mentally retarded; 7. Welfare of the weaker
sections, and in particular, of the Scheduled Castes and the Scheduled Tribes; 8. Public distribution
system; 9. Maintenance of community assets. So, point 2 is correct.

• Directive Principles of State Policy: Article 47 puts a duty on the State to raise the level of
nutrition and the standard of living and to improve public health. So, point 3 is correct.

• Fundamental Duties: There is no direct mention of health in the fundamental duties. So,
point 4 is not correct.

• Fundamental Rights: Right to health is protected under Article 21 of the Constitution. So,
point 5 is correct.

Therefore, option (c) is the correct answer.

Subject: Polity | Constitution

Tags: Conceptual

Q3. Which of the following provisions of the Constitution of India enforce ‘Fraternity’?

1. Article 14

2. Article 15

3. Article 17

4. Article 18

Select the correct answer using the code given below.

a) 1 and 4 only

b) 2 and 3 only

c) 1, 2 and 3 only
d) 1, 2, 3 and 4

Answer: d

Explanation:

• Fraternity refers to brotherhood. It is defined as “brotherhood among disparate bodies


united in their interests, aims, beliefs and so on”. Legal principles like equality and notions like
statehood can be viewed as contrivances for promoting fraternity. Fraternity also suggests "a sense
of belonging to a unit with which one can readily, if not naturally, identify". One of the primary
functions of law is to secure peaceful co-existence of people, which necessitates recognition and
promotion of fraternal feelings among people. The state by subjecting people to the same law seeks
to secure fraternity among people. In order to secure fraternity the law should renounce and also
seek to provide protection against exclusion, discrimination, subordination and oppression.

• In India, the Preamble to the Constitution recognizes fraternity as one of the aims or
principles of the Union of India. There are a number of provisions in the Constitution which seek to
promote fraternity like: Recognition of single citizenship is geared towards instilling a sense of
belongingness among people with respect to subjecting everyone to the same laws (Article 14),
prohibition of discrimination (Article 15), prohibition of titles (Article 18), abolition of untouchability
(Article 17). Even the federal structure established by the Constitution has a unitary bias which
ensures that, irrespective of the cultural, linguistic and other variations, a feeling of commonality is
sought to be maintained.

Therefore, option (d) is the correct answer.

Subject: Polity | Fundamental Rights

Tags: Conceptual

Q4. With reference to the Right to Equality under the Indian Constitution, consider the following
statements:

1. Equality before law implies the right to equality of treatment in equal circumstances.

2. Equal protection of laws forms the basis for the positive discrimination policy of the Government
in India.

3. Equality before Law is a British concept which substantially supports the doctrine of Rule of Law.

Which of the statements given above is/are correct?

a) 1 only

b) 3 only

c) 2 and 3 only

d) 1, 2 and 3

Answer: c

Explanation:

• ‘Equality before law’ is a somewhat negative concept and implies absence of any special
privilege by reason of birth, creed or the like, in favour of any individual or classes etc. On the other
hand, ‘Equal Protection of Law’ is a more positive concept and implies the right to equality of
treatment in equal circumstances. So, statement 1 is not correct.

• Equal protection of laws is an American concept and means that “among equals, the law
should be equal and equally administered, that likes should be treated alike”. This concept requires
affirmative action by the State towards unequal by providing facilities and opportunities. Thus, it
forms the bedrock of the concept of positive discrimination policy in India. So, statement 2 is
correct.

• Equality before law is the second corollary of AV Dicey’s concept of Rule of Law, which has
been borrowed from the British Constitution. This concept has the following three elements or
aspects:

(i) Absence of arbitrary power, that is, no man can be punished except for a breach of law.

(ii) Equality before the law, that is, equal subjection of all citizens (rich or poor, high or low, official or
non-official) to the ordinary law of the land administered by the ordinary law courts.

(iii) The primacy of the rights of the individual, that is, the constitution is the result of the rights of
the individual as defined and enforced by the courts of law rather than the constitution being the
source of the individual rights.

The first and the second elements are applicable to the Indian System and not the third one. In the
Indian System, the constitution is the source of the individual rights. So, statement 3 is correct.

• The Supreme Court held that the ‘Rule of Law’ as embodied in Article 14 is a ‘basic feature’
of the constitution. Hence, it cannot be destroyed even by an amendment

Therefore, option (c) is the correct answer.

Relevance: Recently, Haryana Government challenged in the Supreme Court, an order of Punjab &
Haryana High Court quashing 75% quota for locals in private sector jobs in Haryana and this raised
the question of Equality.

Subject: Polity | Fundamental Rights

Tags: Conceptual

Q5. Which one of the following best defines ‘Amicus Curiae’?

a) A person party to the litigation who volunteers to help the court.

b) A human rights activist who fights for the rights of indigenous people.

c) A person not a party to the litigation who volunteers to help the court.

d) A member of the judicial bar who fights for environmental rights.

Answer: c

Explanation:

• Amicus Curiae is Latin for "friend of the court’’. As per the definition in Merriam Webster
amicus Curiae is defined as -One (such as a professional person or organization) that is not a party to
a particular litigation but that is permitted by the court to advise it in respect to some matter of law
that directly affects the case in question.

• Amicus curiae refers to a person who volunteers to help the court in deciding some matters;
however it is the discretion of the court whether to accept it or not. In certain cases, the court
appoints the amicus curiae, where there is no one to represent one of the parties to the case or
where the question in the case refers to the greater public good.

Therefore, option (c) is the correct answer.

Subject: Polity | Judiciary

Tags: Conceptual

Q6. With reference to the Narcotic Drugs and Psychotropic Substances Act, 1985 (NDPS Act),
consider the following statements:

1. It was enacted to give effect to a directive under Part IV of the Indian Constitution.

2. Use of narcotic drugs for scientific or medicinal purposes is exempted from the premises of the
law.

3. The quantum of punishment is dependent on the amount of psychotropic substances discovered


from accused.

4. An addict convicted for offences involving small quantities of narcotic drugs can be immune from
prosecution if he volunteers for de-addiction.

Which of the statements given above is/are correct?

a) 1 and 2 only

b) 2 only

c) 1, 3 and 4 only

d) 1, 2, 3 and 4

Answer: d

Explanation:

• Narcotic Drugs and Psychotropic Substances have a variety medical and scientific uses.
However, they can be and are abused and trafficked. India's approach towards Narcotic Drugs and
Psychotropic Substances is envisaged in Article 47 of the Constitution of India which mandates that
the: State shall endeavour to bring about prohibition of the consumption except for medicinal
purposes of intoxicating drinks and of drugs which are injurious to health. So, statement 1 is correct.

• The same principle of preventing use of drugs except for medicinal use was also adopted in
the three drug related International Conventions, namely, Single Convention on Narcotic Drugs,
1961, Convention on Psychotropic Substances, 1971 and the United Nations Convention Against
Illicit Traffic in Narcotic Drugs and Psychotropic Substances, 1988. India has signed and ratified the
above-mentioned conventions.
• The Narcotic Drugs and Psychotropic Substances Act, 1985 Act was enacted taking into
account India's obligations under the three UN drug Conventions as well as Article 47 of the
Constitution.

Narcotic Drugs and Psychotropic Substances Act, 1985 (NDPS Act):

• This Act prohibits, a person from the production/manufacturing/cultivation, possession,


sale, purchasing, transport, storage, and/or consumption of any narcotic drug or psychotropic
substance, except for medical or scientific purposes.

• Items defined as narcotics under the act include coca leaf, cannabis, opium as well as poppy
straw.

• Any use of narcotic drugs and psychotropic substances for scientific or medicinal purposes
are exempted from the rigor of the law. So, statement 2 is correct.

• The quantum of punishment under NDPS Act depends upon the quantity of narcotic drugs
and psychotropic substances found. So, statement 3 is correct.

• An addict convicted for consumption under the act or for offenses involving small quantities
will be immune from prosecution if he volunteers for de-addiction. So, statement 4 is correct.

Therefore, option (d) is the correct answer.

Relevance: Narcotics Control Bureau has been arresting many high profile people under the Narcotic
Drugs and Psychotropic Substances Act, 1985 (NDPS Act).

Subject: Polity| Bills and Acts

Tags: Factual

Q7. Who among the following presides over the Lok Sabha if the office of both Speaker and Deputy
Speaker falls vacant?

a) A Member so appointed by the President

b) A Member elected by the Lok Sabha

c) Any Member of Panel of Chairpersons

d) Senior-most Member of the Lok Sabha

Answer: a

Explanation:

• The Deputy Speaker presides over the Lok Sabha when the Speaker is absent from the sitting
of the House.

• In case of vacancy: When the Offices of both the Speaker and the Deputy Speaker fall
vacant, the duties of the Office of the Speaker are performed by such Member of the Lok Sabha as
the President may appoint for the purpose. The person so appointed is known as the Speaker pro
tem.

• In case of absence: The Rules of Procedure and Conduct of Business in Lok Sabha provide
that at the commencement of the House or from time to time, as the case may be, the Speaker shall
nominate from amongst the Members a Panel of not more than ten Chairpersons, any one of whom
may preside over the House in the absence of the Speaker and the Deputy Speaker when so
requested by the Speaker or, in the absence of the Speaker, by the Deputy Speaker.

Therefore, option (a) is the correct answer.

Subject: Polity | Parliament

Tags: Conceptual

Q8. Consider the following statements:

1. The power to create new districts or alter or abolish existing districts rests with the respective
State Government.

2. Renaming of a city or town by a State is subjected to the consent of the Central Government.

3. States have the unilateral power to change their respective names and require no prior
Parliamentary approval.

Which of the statements given above is/are correct?

a) 1 and 2 only

b) 2 only

c) 3 only

d) 1, 2 and 3

Answer: a

Explanation:

• The power to create new districts or alter or abolish existing districts rests with the State
governments. This can either be done through an executive order or by passing a law in the State
Assembly. Many States prefer the executive route by simply issuing a notification in the official
gazette. So, statement 1 is correct.

• Executive instructions have been issued by the Union to all State Governments regarding the
procedure to be adopted for renaming villages/ towns/ cities etc. These are also applicable for
renaming of a Railway Station. The proposals for renaming a town/ city/ village/ railway station are
to be submitted by the State Governments to the Central Government for approval. Though the
Central Government accords approval, Gazette Notification giving effect to the change is issued by
the State Government concerned. The Home Ministry is the ministry concerned in this respect. So,
statement 2 is correct.

• The renaming of states is governed by Article 3 of the Constitution of India. The Article
empowers the Parliament to alter the name of any State by law. The renaming requires
Parliamentary approval under Article 3 and 4 of the Constitution, and the President has to refer the
same to the relevant state legislature for its views. Thus, states have no unilateral power to change
the name of their states and require prior parliamentary approval for the same. So, statement 3 is
not correct.

Therefore, option (a) is the correct answer.

Relevance: Recently, the state of Punjab created the 32nd District of Malerkotla.
Subject: Polity |Federalism

Tags: Conceptual, Factual

Q9. With reference to the Press Council of India, consider the following statements:

1. It is neither a constitutional nor a statutory body.

2. Its Chairman is nominated by the Chief Justice of India.

3. It is funded by revenue collected as fees levied on the registered newspapers.

4. Its jurisdiction includes print media, electronic media and TV news channels.

Which of the statements given above is/are correct?

a) 1 and 2 only

b) 3 only

c) 1 and 4 only

d) 2 and 4 only

Answer: b

Explanation:

• The Press Council of India was first constituted on 4th July, 1966 as an autonomous,
statutory, quasi-judicial body. The Council set up under the Act of 1965 functioned till December
1975. During the Internal Emergency, the Act was repealed and the Council abolished w.e.f.
1/1/1976. The current Press Council has been established by the Press Council Act, 1978. So,
statement 1 is not correct.

• Under the original Act of 1965, the Chairman was nominated by the Chief Justice of India.
However, under Section 5 of the 1978 Act, the Chairperson of the council is nominated by a
Committee consisting of the Chairman of the Council of States (Rajya Sabha), the Speaker of the
House of the People (Lok Sabha) and a person elected by the members of the Council. So, statement
2 is not correct.

• The Act states that the Council shall have its own fund, and the fees collected by it, all such
sums as may, from time to time, be paid to it by the Central Government and all grants and advances
made to it by any other authority or person shall be credited to the Fund and all payments by the
Council shall be made therefrom. The Council is funded by revenue collected by it as fee levied on
the registered newspapers in the country on the basis of their circulation. So, statement 3 is correct.

• Electronic media, TV news channels, social media like Whatsapp/Twitter/Facebook do not


come under the jurisdiction of the Press Council of India. Complaints against and by the print media
are adjudicated by the Council by adhering to the Press Council (Procedure for Inquiry) Regulations,
1979. So, statement 4 is not correct.

Therefore, option (b) is the correct answer.

Relevance: A fact finding committee (FFC) of the Press Council of India (PCI) has found that the
“news media in the Jammu & Kashmir region, and especially in the Valley is slowly being choked
mainly because of the extensive curbs imposed by the local administration”.
Subject: Polity | Non-Constitutional Bodies

Tags: Factual

Q10. Which one of the following best defines the term ‘law’?

a) Political norms established by the society to regulate human behaviour

b) Rules made by the designated political authority having legitimacy to do so

c) Rules that govern human behaviour and are not coercive in nature

d) Social norms established by the society to ensure justice to the deprived

Answer: b

Explanation:

• Law means rules made by the superior political or judicial authority (comprising human
beings) having legitimate power to make the rules for the guidance of the conduct of other human
beings.

• These are the norms not established by the society, but legitimate authority (either political
or judicial). The law tries to establish norms in the society to regulate human behaviour. The norms
may be social, political, economic or environmental. Norms established by law are coercive in
nature.

Therefore option (b) is the correct answer.

Subject: Polity | Miscellaneous(pol)

Tags: Conceptual

Q11. Which of the following is/are not defined in the Constitution of India?

1. Religious Minority

2. Anglo Indian

3. Untouchability

4. Martial Law

Select the correct answer using the code given below.

a) 1 and 2 only

b) 1, 3 and 4 only

c) 2 only

d) 3 and 4 only

Answer: b

Explanation:

• The Constitution of India mentions various terms but has not defined them thus leaving it up
to the courts to interpret the meaning of those terms. Article 29 and 30 of the Constitution of India
provide special rights to linguistic and religious minorities. However, the word minority has not been
defined anywhere by the Constitution. The Government of India has declared Muslims, Jains, Sikhs,
Buddhists and Zoroastrians as the religious minorities in India. But there is no clarity on the
definition of the word “minority”. So, point 1 is correct.

• The term Anglo Indian has been defined by the Constitution under Article 366(2) to mean a
person whose father or any male progenitors is or was of European descent but who is domiciled
within the territory of India and is or was born within such territory. So, point 2 is not correct.

• Article 17 prohibits untouchability. There is no definition of the meaning of Untouchability


either in the Constitution nor is it mentioned in the Protection of Civil Rights Act, 1955. So, point 3 is
correct.

• Article 34 of the Constitution of India gives Parliament the power to indemnify, by law, any
person in service of Union or State or any other person with respect to acts done during operation of
martial law in an area. Martial law has not been defined in the Constitution. Moreover, there is no
provision in the Constitution authorising proclamation of martial law. So, point 4 is correct.

Therefore, option (b) is the answer.

Relevance: The term Minority was recently in news due to a petition filed in the Supreme Court
which prayed for declaration of Hindus as minorities in certain states of India.

Subject: Polity | Constitution

Tags: Conceptual, Factual

Q12. Consider the following statements:

1. Both Ganges River dolphins and Irrawaddy dolphins have a gestation period of more than one
year.

2. Indus and Ganges River dolphins are found in freshwater whereas Irrawaddy dolphins can be
found in brackish water.

3. Unlike Indus River dolphins, Ganges River dolphins are functionally blind.

4. Indus River Dolphins, Ganges River Dolphins and Irrawaddy Dolphins species are listed as
Endangered on the IUCN Red List of Threatened Species.

Which of the statements given above are correct?

a) 1, 2 and 4 only

b) 2 and 4 only

c) 1 and 3 only

d) 1, 2, 3 and 4

Answer: b

Explanation:

Ganges River Dolphin:


• The Ganges River Dolphin (Platanista gangetica) is a mammal primarily found in the Ganges
and Brahmaputra Rivers and their tributaries in India, Bangladesh and Nepal. The Ganges river
dolphin can only live in freshwater and is essentially blind. They are frequently found alone or in
small groups, and generally a mother and calf travel together.

• Calves have been observed between January and May and do not appear to stay with the
mother for more than a few months. Gestation is thought to be approximately 9-10 months. (For
Irrawaddy Dolphins the gestation is about 14 months) Females are larger than males and give birth
once every two to three years to only one calf. So, statement 1 is not correct.

• Ganges River Dolphin is listed in Appendix I of CITES (Convention on International Trade in


Endangered Species of Wild Fauna and Flora), Endangered in IUCN red list and Schedule I of India’s
Wildlife (Protection), Act, 1972.

Indus river dolphin:

• Indus river dolphins are believed to have originated in the ancient Tethys Sea. When the sea
dried up approximately 50 million years ago, the dolphins were forced to adapt to its only remaining
habitat, rivers. Until recently, it was believed that these dolphins were endemic to Pakistan. But in
2007, a remnant but viable population of Indus dolphins was discovered in Punjab’s Harike wildlife
sanctuary and in the lower Beas river.

• These dolphins are usually found individually or in pairs but have been seen in groups of up
to 10.

• They have adapted to life in the muddy river and are functionally blind (Like the Ganges
River Dolphin). They rely on echolocation to navigate, communicate and hunt prey including prawns,
catfish, and carp. The Indus river dolphin is classified as endangered by the International Union for
the Conservation of Nature (IUCN). So, statement 3 is not correct.

Irrawaddy dolphins:

• Irrawaddy dolphins are patchily distributed throughout freshwater and coastal areas in
Southeast Asia. The three main freshwater populations inhabit the Ayeyarwady, Mekong, and
Mahakam rivers. Coastal populations live in brackish or saline waters, and are generally associated
with areas of freshwater input, such as river deltas, mangrove channels, and estuaries. So, statement
2 is correct.

• They tend to occur in small groups of 2-6 individuals.

• Gestation is thought to be about 14 months, with births peaking in the pre-monsoon season
of April-June, but they may occur year-round.

• The Irrawaddy dolphin species, the Gangetic River Dolphin and the Indus Valley Dolphin are
red-listed as Endangered on the IUCN Red List of Species. So, statement 4 is correct.

Therefore, option (b) is the correct answer.

Relevance: The Union Environment ministry has designated October 5 as the National Dolphin Day

Subject: Environment | Biodiversity & Conservation

Tags: Conceptual, Factual


Q13. With reference to mangroves, consider the following statements:

1. They prevent salinity and seawater ingress into the inland agricultural areas.

2. Barnacles and crabs promote their growth by attaching themselves to the young

seedlings.

3. Growth of the shrimp sector in India has helped in their conservation.

4. State of West Bengal followed by Gujarat has the largest mangrove cover in India.

Which of the statements given above are correct?

a) 1 and 4 only

b) 2 and 3 only

c) 1, 2 and 4 only

d) 3 and 4 only

Answer: a

Explanation:

• Mangroves are plant communities of the tropical and subtropical intertidal coastal zone.
Mangroves ecosystems are known to provide variety of services and goods to mankind. It provides
timber and non-timber forest products, medicines, fisheries, recreation, ecotourism, bio-filtration,
nursery grounds, coastal protection, and carbon storage and sequestration. Mangrove forests are
considered to be the most productive ecosystems with high biomass especially below-ground.
Productive, protective, economical and social functions mangrove ecosystems provide us reason to
conserve and manage them sustainably. Mangroves also play a very significant role in maintaining
the coastal environment, reducing the impact of wave action and erosion in the coastal areas,
preventing salinity and seawater ingress into the inland agricultural areas, and also providing
protection to the coastline from the impact of cyclones. Apart from these ecological functions,
mangroves play a very significant economic role in the lives of the coastal village communities. The
villagers are dependent on mangroves mainly for fodder, fuel-wood and fishing activities. So,
statement 1 is correct.

Natural hazards and threats to mangroves in India:

• Cyclones, typhoons and strong wave action, especially in the geographically vulnerable
Andaman and Nicobar Islands.

• Browsing and trampling by wildlife and livestock (goats, buffaloes and cows), which are
often left to graze freely, especially in areas close to human habitation.

• Infestation by barnacles which attach to young seedlings, interfering with respiration and
photosynthesis and delaying seedling growth. So, statement 2 is not correct.

• Damage by oysters to the young leaves and plumules of Rhizophora and Ceriops plants.

• Crabs, which attack young seedlings, girdle the root collars and eat the fleshy tissues of the
propagules.
• Weeds such as Acrostichum aureum and Acanthus species, which often occupy deforested
mangrove areas and restrict the re-growth of economic mangrove tree species.

• Impact of Shrimp sector: The fast development of the shrimp sector required the conversion
of flat, coastal lands to shrimp ponds. Part of the shrimp pond construction took place in mangroves,
and shrimp aquaculture has been an important cause of the conversion of mangroves in India.
Mangrove conversion has been undertaken by both small-scale extensive farms and by larger-scale
semi-intensive and intensive farms. So, statement 3 is not correct.

• In India, mangroves are found in the States of West Bengal, Orissa, Andhra Pradesh, Tamil
Nadu, Andaman and Nicobar Islands, Kerala, Goa, Maharashtra, Karnataka and Gujarat. The
Mangroves cover in Gujarat are second only to the Mangroves in the West Bengal on the East Coast
in terms of area. So, statement 4 is correct.

Therefore, option (a) is the correct answer.

Relevance: Jawaharlal Nehru Port Trust (JNPT) will transfer 815 hectares of mangrove land under its
jurisdiction to the forest department that will be declared as reserved forests. Similarly, the Mumbai
Metropolitan Regional Development Authority (MMRDA) will hand over 199 hectares to the
department.

Subject: Environment | Biodiversity & Conservation

Tags: Factual

Q14. Which of the following creatures is/are venomous in nature?

1. Komodo dragon

2. Jellyfish

3. Python

Select the correct answer using the code given below.

a) 1 only

b) 1 and 2 only

c) 2 only

d) 3 only

Answer: b

Explanation:

Komodo dragon:

• Komodo dragons are the largest extant lizard species. The dragon is a monitor lizard of the
family Varanidae. It occurs on Komodo Island and a few neighbouring islands of the Lesser Sunda
Islands of Indonesia. The popular interest in the lizard’s large size and predatory habits has allowed
this endangered species to become an ecotourist attraction, which has encouraged its protection.

• More than a decade ago, researchers have discovered that the bite of the Komodo dragon is
venomous. Magnetic resonance imaging scans revealed complex venom glands in the dragons'
mouths that had never been documented before. So, point 1 is correct.

Jellyfish:

• These simple invertebrates are members of the phylum Cnidaria, which includes creatures
such as sea anemones, sea whips, and corals. Jellyfish are about 95 percent water. Lacking brains,
blood, or even hearts, jellyfish are pretty simple critters. They are composed of three layers: an
outer layer, called the epidermis; a middle layer made of a thick, elastic, jelly-like substance called
mesoglea; and an inner layer, called the gastrodermis.

• The long tentacles trailing from the jellyfish body can inject a person with venom from
thousands of microscopic barbed stingers. Jellyfish stings vary greatly in severity. Most often they
result in immediate pain and red, irritated marks on the skin. So, point 2 is correct.

Python:

• Pythons are non-venomous snakes found in Asia, Africa and Australia. Because they are not
native to North or South America, they are considered Old World snakes. The word python can refer
to both the family Pythonidae and the genus Python, found within Pythonidae. There are 41 species
of python found within the family Pythonidae. So, point 3 is not correct.

• Most Pythons are large, with the reticulated python (Python reticulatus) of Asia attaining a
maximum recorded length of 9.6 meters (31.5 feet).

Therefore, option (b) is the correct answer.

Relevance: The Indian Venom Research Institute (IVRU), will be set up in association with the
Evolutionary Venomics Lab under the Indian Institute of Science (IISc) in Bengaluru, Karnataka. The
centre will also have a serpentarium which will house around 500 snakes belonging to about 23
species, besides other venomous creatures, including scorpions and spiders.

Subject: Environment | Biodiversity & Conservation

Tags: Factual

Q15. Consider the following statements:

1. Pollinators help carry pollen from the female part of the flower to the male part of the same or
another flower.

2. Pollutants can react with volatile organic compounds released by flowers and change the scent of
flowers.

3. Cultivation of Genetically Modified Organisms and mono-cropping causes decline in the number
of pollinators.

4. Air pollution can cause behavioural, physiological and genetic changes in wild bees.

Which of the statements given above are correct?

a) 1 and 2 only
b) 1 and 3 only

c) 2, 3 and 4 only

d) 1, 2, 3 and 4

Answer: c

Explanation:

Pollutants and pollinators:

• A pollinator is anything that helps carry pollen from the male part of the flower (stamen) to
the female part of the same or another flower (stigma). The movement of pollen must occur for the
plant to become fertilized and produce fruits, seeds, and young plants. So, statement 1 is not
correct.

• Flowers release odors as chemicals called volatile organic compounds, which help insects
locate flowers. Pollutants could react with flowers and change the scents of flowers, making them
harder to find. So, statement 2 is correct.

• There are several causes for the decline in the number of pollinators. Most of them are the
result of an increase in human activities:

o Land-use change and fragmentation

o Changes in agricultural practices including use of chemical pesticides, fungicides and


insecticides

o Change in the cropping pattern and crops like the cultivation of Genetically Modified
Organisms (GMOs) and mono-cropping. So, statement 3 is correct.

o High environmental pollution from heavy metals and nitrogen

o Growth of invasive alien species

• Research suggested that not all insects are equally impacted by pollutants. Beetles and
parasitic wasps appeared resilient to pollutants. The researchers suspect that this could be because
the above insects relied on visual cues rather than odour to reach flowers. Impact on bees: A study
shows that air pollution causes behavioural, physiological and genetic changes in wild bees.
Pollutant impact on bees is observed even in areas well within the current national standards for
Particulate Matter pollution. Pollution impacts survival rates among bees exposed to air pollution
and could harm pollination services in forest or agrarian ecosystems near urban areas. So, statement
4 is correct.

Therefore, option (c) is the correct answer.

Relevance: Recent reports have stated that instability in yield of pollinator-dependent crops is a
‘serious’ or ‘high risk’ in Latin America, Asia, Africa and Oceania.

Subject: Environment | Pollution

Tags: Conceptual, Factual


Q16. ‘Oxybenzone, Octocrylen and Octinoxate’ considered as the Toxic ’O3s’, are most likely present
in which of the following?

a) Varnish and paints

b) Insecticides

c) Food preservatives

d) Sunscreens

Answer: d

Explanation:

Oxybenzone, octocrylene, and octinoxate:

• Oxybenzone, octinoxate and octocrylene are active ingredients present in more than two-
thirds of all sunscreens. They pose a threat to public health, marine life and coral reefs.

• Octocrylene in sun protection products degrades into benzophenone, a carcinogen that can
also interfere with key hormones and reproductive organs.

• The “Toxic 3 Os” have been shown to destroy coral and trigger health risks to people and
marine life. They cause human cell damage that has been linked to cancer, disrupt hormones, have
been found in breast milk, blood and urine and are known allergens.

• These chemicals are also devastating to coral reefs and marine life. They wash off people’s
bodies when they swim and contaminate through wastewater runoff and cause ‘zombie’ coral which
looks healthy but is unable to reproduce, coral bleaching as well as other issues.

• Oxybenzone is particularly toxic to corals at concentrations as low as a few parts per trillion,
the equivalent of three drops in an Olympic-size swimming pool may be enough to severely damage
or kill coral.

Therefore, option (d) is the correct answer.

Relevance: Oxybenzone, octinoxate and octocrylene pose a threat to public health, marine life and
coral reefs, according to latest research

Subject: Environment |Pollution

Tags: Factual

Q17. Which of the following are the sources of dioxin pollution?

1. Production of herbicides

2. Waste incineration

3. Cigarette smoke

4. Chlorine bleaching of pulp and paper

Select the correct answer using the code given below.


a) 1 and 3 only

b) 2, 3 and 4 only

c) 1, 2 and 4 only

d) 1, 2, 3 and 4

Answer: d

Explanation:

• Dioxins are called persistent organic pollutants (POPs), meaning they take a long time to
break down once they are in the environment. Dioxins are highly toxic and can cause cancer,
reproductive and developmental problems, damage to the immune system, and can interfere with
hormones.

• Dioxins are found throughout the world in the environment, and they accumulate in food
chains, concentrating mainly in the fatty tissue of animals. Some sources of dioxins:

o Industrial activities: Dioxin is not produced or used commercially in the United States. It is a
contaminant formed during the production of some chlorinated organic compounds, including a few
herbicides such as Silvex.

o Burning: Combustion processes such as waste incineration (commercial or municipal) or


burning fuels (like wood, coal or oil) form dioxins.

o Bleaching: Chlorine bleaching of pulp and paper and other industrial processes can create
small quantities of dioxins in the environment.

o Smoking: Cigarette smoke also contains small amounts of dioxins.

Therefore, option (d) is the correct answer.

Subject: Environment |Pollution

Tags: Factual

Q18. In which of the following species, does the male carry the fertilized eggs and nurture
developing young ones?

a) Pufferfish

b) Oyster

c) Seahorse

d) Galapagos Green Turtle

Answer: c

Explanation:

• Seahorse (genus Hippocampus) is any of about 50 species of marine fishes allied to


pipefishes in the family Syngnathidae. Seahorses are found in shallow coastal waters in latitudes
from about 52° N to 45° S. Their habitats include coral reefs, mangroves, seagrass beds, and
estuaries.

Reproductive behaviour of seahorses:

• The reproductive behaviour of seahorses is notable in that the male carries the fertilized
eggs. After an elaborate courtship, the female uses an ovipositor (egg duct) to place her eggs into a
brood pouch located at the base of the male’s tail where the eggs are later fertilized.

• Depending on the species, the eggs remain in the pouch between 10 days and six weeks.
During this time the male nurtures the developing young by regulating the chemistry of the fluid
inside the pouch, slowly transforming it from that of his internal body fluids to that of saltwater as
pregnancy progresses.

• To nourish the growing young, the male also produces inorganic compounds and releases
the hormone prolactin, which helps break down the proteins contributed by the female. Once the
eggs hatch, the male convulses his body and expels the young through a single opening in the pouch.

Therefore, option (c) is the correct answer.

Relevance: A paper published last year showed that between 4.98 and 13.64 million seahorses land
as by-catch from India’s southeast coast every year. Hippocampus kuda and H. trimaculatus are the
two species of sea horse that are vulnerable to overfishing.

Subject: Environment | Biodiversity & Conservation

Tags: Factual

Q19. Consider the following statements:

1. Pre-independence, the Governor-General of India was the ex-officio Chancellor of the State
Universities of Calcutta, Bombay and Madras.

2. At present, the Governor of a state can be appointed as the ex-officio Chancellor of the respective
State University.

Which of the statements given above is/are correct?

a) 1 only

b) 2 only

c) Both 1 and 2

d) Neither 1 nor 2

Answer: b

Explanation:

• The practice of appointing the governor as ex-officio chancellor of universities dates to the
pre-Independence era. The foundation was laid down in the dispatch of Sir Charles Wood in 1854 to
the court of directors described as ‘The Magna Carta of English Education’ in India. This document
led the court of directors to deliberate that it was perhaps time to establish Indian universities
modelled on the London University. Consequently, the universities of Calcutta, Bombay and Madras
were established in 1857. These universities like the university back in London consisted of a
chancellor, vice-chancellor and the senate. The governor-general of British India was the chancellor
of the Calcutta University and the Governors of Bombay and Madras headed their State Universities.
So, statement 1 is not correct.

• After independence, whenever a State University is established, it is done with the help of a
legislation passed by the state assembly. The statute unequivocally incorporates this vestigial
provision of making the governor of that state its ex-officio chancellor. And this is why the governor,
till date, exercises the power to appoint vice-chancellors. However, what we need to note is that
post-Independence, the governor is a Constitutional functionary. So, statement 2 is correct.

Therefore, option (b) is the correct answer.

Relevance: Recently, the Legislative Assembly of Tamil Nadu has passed two bills that seek to
transfer the Governor’s power in appointing Vice-Chancellors of 13 state universities to the state
government.

Subject: Polity | Governor

Tags: Factual

Q20. In the context of evolution of Civil Services during the colonial rule in India, consider the
following statements:

1. Merit based modern Civil Service in India was introduced on the recommendations of Lord
Macaulay’s Report of the Indian Civil Services.

2. Indian Civil Service Examination began to be held in India only after independence.

3. Federal Public Service Commission was established under the Government of India Act, 1919.

Which of the statements given above is/are correct?

a) 1 only

b) 2 and 3 only

c) 3 only

d) None

Answer: a

Explanation:

• Civil Servants for the East India Company used to be nominated by the Directors of the
Company and thereafter trained at Haileybury College in London and then sent to India. Following
Lord Macaulay’s Report of the Select Committee of British Parliament, the concept of a merit-based
modern Civil Service in India was introduced in 1854. The Report recommended that patronage-
based system of East India Company should be replaced by a permanent Civil Service based on a
merit-based system with entry through competitive examinations. For this purpose, a Civil Service
Commission was set up in 1854 in London and competitive examinations were started in 1855. So,
statement 1 is correct.
• Initially, the examinations for Indian Civil Service were conducted only in London. The
maximum age was 23 years and the minimum age was 18 years. The syllabus was designed such that
European Classics had a predominant share of marks. All this made it difficult for Indian candidates.
Nevertheless, in 1864, the first Indian, Shri Satyendranath Tagore, brother of Shri Rabindaranath
Tagore succeeded. Three years later 4 other Indians succeeded. Throughout the next 50 years,
Indians petitioned for simultaneous examinations to be held in India without success because the
British Government did not want many Indians to succeed and enter the ICS. It was only after the
First World War and the Montagu Chelmsford reforms that this was agreed to. From 1922 onwards
the Indian Civil Service Examination began to be held in India also, first in Allahabad and later in
Delhi with the setting up of the Federal Public Service Commission. The Examination in London
continued to be conducted by the Civil Service Commission. So, statement 2 is not correct.

• The Government of India Act, 1919 provided for the establishment in India of a Public
Service Commission which should “discharge, in regard to recruitment and control of the Public
Services in India, such functions as may be assigned thereto by rules made by the Secretary of State
in Council”. The Lee Commission, in their report in the year 1924, recommended that the statutory
Public Service Commission contemplated by the Government of India Act, 1919 should be
established without delay. Subsequent to the provisions of Section 96(C) of the Government of India
Act, 1919 and the strong recommendations made by the Lee Commission in 1924 for the early
establishment of a Public Service Commission, it was on October 1, 1926 that the Public Service
Commission was set up in India for the first time.

• Further, the Government of India Act, 1935 envisaged a Public Service Commission for the
Federation and a Provincial Public Service Commission for each Province or group of Provinces.
Therefore, in terms of the provisions of the Government of India Act, 1935 and with its coming into
effect on 1st April, 1937, the Public Service Commission became the Federal Public Service
Commission. So, statement 3 is not correct.

Therefore, option (a) is the correct answer.

Subject: Polity | Constitutional Bodies

Tags: Factual

Q21. Which of the following can be the best reason to call the Lok Sabha in India a ‘Popular
Chamber’?

a) It is composed of nominated members having special knowledge in literature, science, art and
social service

b) It is composed of representatives having lucid knowledge of politics and governance

c) It is composed of representatives chosen by direct election on the basis of adult franchise

d) It is composed of representatives who are popular among the people of their constituencies

Answer: c

Explanation:

• Parliament of India consists of three parts viz, the President, the Council of States and the
House of the People. The Lok Sabha is the Lower House (First Chamber or Popular House) which
represents the people of India as a whole. It has representation from the states as well as union
territories.
• The Lok Sabha is composed of representatives of the people chosen by direct election on the
basis of adult suffrage. That is why it is called the popular chamber.

Therefore option (c) is the correct answer.

Subject: Polity | Parliament

Tags: Conceptual

Q22. Consider the following statements:

1. Both India and the USA have an integrated judicial system to help establish a strong federation.

2. The age of retirement of the Judges in both the American and Indian Supreme Court is the same.

Which of the statements given above is/are correct?

a) 1 only

b) 2 only

c) Both 1 and 2

d) Neither 1 nor 2

Answer: d

Explanation:

• Unlike the American Constitution, the Indian Constitution has established an integrated
judicial system with the Supreme Court at the top and the high courts below it. Under a high court
(and below the state level), there is a hierarchy of subordinate courts, that is, district courts and
other lower courts. This single system of courts, adopted from the Government of India Act of 1935,
enforces both Central laws as well as state laws. On the other hand America doesn’t have an
integrated judicial system. The United States is a federal system, with a central federal government
and individual governments for each of the fifty states. As with the other branches of government,
each of the states has their own complete judicial system (state courts) as does the United States
itself (federal courts). Each judicial system has a number of courts of original jurisdiction, in which
cases are originally filed and tried. The jurisdiction of these trial courts can be both geographically
and subject matter based. Each system also has a smaller number of intermediate appellate courts.
These courts hear appeals from the trial courts. An appeal is a claim by the losing party that the
lower court has made a mistake of law. Usually, a losing party is entitled to one appeal as a matter
of right. Each court system also has a supreme court, which hears appeals from the appellate courts.
Appeals to the Supreme Court are usually discretionary, that is the court may choose whether or not
to hear the appeal. So, statement 1 is not correct.

• The United States of America is unique in the democratic world with respect to its tenure of
judges of the Federal court who enjoy lifelong tenures. Judges of the American Supreme Court do
not have a fixed tenure and continue in office for life. The Indian Supreme Court Justices, on the
other hand, retire at the age of 65 years. So, statement 2 is not correct.

Therefore, option (d) is the correct answer.


Relevance: Recently, the American President, Joe Biden nominated the first Black Woman to the
Supreme Court of the United States.

Subject: Polity | Constitution

Tags: Conceptual, Factual

Q23. In the context of Indian polity, which one of the following is the most appropriate definition of
‘Freedom’?

a) Complete absence of restraint

b) Opportunity to do whatever one likes

c) Protection against the tyranny of State

d) Liberty with reasonable restrain

Answer: d

Explanation:

• Freedom is said to exist when external constraints on the individual are absent. In terms of
this definition an individual could be considered free if he/she is not subject to external controls or
coercion and is able to make independent decisions and act in an autonomous way. However,
absence of constraints is only one dimension of freedom. Freedom is also about expanding the
ability of people to freely express themselves and develop their potential. Freedom in this sense is
the condition in which people can develop their creativity and capabilities. Both these aspects of
freedom — the absence of external constraints as well as the existence of conditions in which
people can develop their talents — are important. A free society would be one which enables all its
members to develop their potential with the minimum of social constraints.

• No individual living in society can hope to enjoy total absence of any kind of constraints or
restrictions. It becomes necessary then to determine which social constraints are justified and which
are not, which are acceptable and which should be removed. To be free means to reduce or
minimise social constraints that limit our ability to make choices freely. However, this is only one
aspect of freedom. In the constitutional discussions in India, the term used for such justifiable
constraints is ‘reasonable restrictions’. The restrictions may be there but they must be reasonable,
i.e., capable of being defended by reason, not excessive, not out of proportion to the action being
restricted, since then it would impinge on the general condition of freedom in society. We must not
develop a habit of imposing restrictions since such a habit is detrimental to freedom.

• Freedom, in this sense, allows the full development of the individual’s creativity, sensibilities
and capabilities: be it in sports, science, art, music or exploration.

Therefore, option (d) is the correct answer.

Subject: Polity | Miscellaneous(pol)

Tags: Conceptual, Factual


Q24. With reference to ‘Financial Emergency’ in India, consider the following statements:

1. This provision has been borrowed from the Weimar Constitution.

2. All money bills passed by state legislature shall be reserved for the consideration of the President
during its operation.

3. There is no maximum period prescribed for its operation.

4. The proclamation for revocation of the emergency by the President doesn’t need parliamentary
approval.

Which of the statements given above are correct?

a) 1 and 3 only

b) 2 and 4 only

c) 1, 2 and 3 only

d) 1, 2, 3 and 4

Answer: d

Explanation:

• The Constitution of India envisages three types of emergencies, namely:

o National emergency on the ground of war or external aggression or armed rebellion16


(Article 352)

o State emergency (President’s Rule) on the ground of failure of Constitutional machinery in


the states (Article 356) or failure to comply with the directions of the Centre (Article 365)

o Financial emergency on the ground of threat to the financial stability or credit of India
(Article 360). This provision has been borrowed from ‘Weimar Constitution’ of Germany. So,
statement 1 is correct.

• While the proclamation of financial emergency (under Article 360) is in operation, the
executive authority of the union/Center can give directions to the states:

o to observe the specified canons of financial propriety;

o to reduce the salaries and allowances of all class of persons serving in the state; and

o to reserve all money bills and other financial bills for the consideration of the President. So,
statement 2 is correct.

• A resolution approving the proclamation of financial emergency can be passed by either


House of Parliament only by a simple majority, that is, a majority of the members of that house
present and voting. Once approved by both the Houses of Parliament, the Financial Emergency
continues indefinitely till it is revoked. This implies two things: there is no maximum period
prescribed for its operation; and repeated parliamentary approval is not required for its
continuation. So, statement 3 is correct.

• No Financial Emergency has been declared so far, though there was a financial crisis in 1991.
The salaries, allowances, privileges, leave and pension of the judges of the Supreme Court are
determined from time to time by the Parliament. They cannot be changed to their disadvantage
after their appointment except during a financial emergency.

• A proclamation declaring financial emergency must be approved by both the Houses of


Parliament within two months from the date of its issue. Once approved by both the Houses of
Parliament, the Financial Emergency continues indefinitely till it is revoked. A resolution approving
the proclamation of financial emergency can be passed by either House of Parliament only by a
simple majority, that is, a majority of the members of that house present and voting. A proclamation
of Financial Emergency may be revoked by the president at any time by a subsequent proclamation.
Such a proclamation does not require the parliamentary approval. So, statement 4 is correct.

Therefore, option (d) is the correct answer.

Subject: Polity | Emergency Provisions

Tags: Factual

Q25. With reference to the Custodial Deaths in India, consider the following statements:

1. Registration of First Information Report (FIR) in a case of custodial death is

mandatory.

2. Inquiry into the cause of custodial deaths can be done by a Judicial Magistrate only.

3. The National Human Rights Commission has set a six-month deadline for the completion of the
Magistrate's enquiry in case of custodial deaths.

Which of the statements given above is/are not correct?

a) 1 only

b) 3 only

c) 1 and 2 only

d) 2 and 3 only

Answer: d

Explanation:

• Custodial death is a cruel form of abuse, that mostly happen where police officer uses
unnecessary force, ultra vires of their powers and torture convicts beyond the limits to take the life
of convicted person. Custodial Death is that concept where any arrested person died in police
custody due to the torture done by any police or officer in charge of such custody, the custody may
be police custody or magistrate custody. As per the present legislation in India, the custodial torture
is the grave result of death, rape and physical torture.

• Registration of FIR in a case of custodial death is mandatory. The Supreme Court clarified in
the decision Lalita kumara vs State of UP (2014) that 'registration of FIR is mandatory, if the
information discloses commission of a cognizable offense and no preliminary inquiry is permissible in
such a situation'. So, statement 1 is correct.
• The law envisaged a process of parallel Magisterial Inquiry immediately after the incident.
Section 176(1) CrPC states that a Magistrate, who is empowered to hold inquiries with respect to an
unnatural death, may hold an inquiry into the cause of death in addition to the investigation held by
the police officer. This is only a general, empowering provision, giving Magistrate the discretion to
hold such an inquiry. Another fact to be noted is that such inquiry can be held either by an Executive
Magistrate or Judicial Magistrate. So, statement 2 is not correct.

• The National Human Rights Commission has also issued guidelines for Magisterial Enquiry. It
has also set a two-month deadline for the completion of an inquiry by the Magistrate. So, statement
3 is not correct.

Therefore, option (d) is the answer.

Relevance: Recently, a report stated that only 21 cases of custodial deaths have resulted in
disciplinary action in the last five years.

Subject: Polity| Bills and Acts

Tags: Factual

Q26. With reference to ‘layers of the Earth’s Atmosphere’, which one of the following statements is
correct?

a) Going up from the troposphere to the mesosphere, the temperature decreases continuously.

b) Going up from the mesosphere to the ionosphere, the temperature increases continuously.

c) Going up from ionosphere to exosphere, the temperature increases continuously.

d) Going up from troposphere to stratosphere, the temperature increases continuously.

Answer: c

Explanation:

• The atmosphere consists of different layers with varying density and temperature. Density is
highest near the surface of the earth and decreases with increasing altitude. The column of
atmosphere is divided into five different layers depending upon the temperature condition. They
are: troposphere, stratosphere, mesosphere, thermosphere and exosphere

• The troposphere is the lowermost layer of the atmosphere. Its average height is 13 km and
extends roughly to a height of 8 km near the poles and about 18 km at the equator. The
temperature in this layer decreases at the rate of 1°C for every 165m of height. This is the most
important layer for all biological activity. The zone separating the troposphere from stratosphere is
known as the tropopause. The temperature here is nearly constant, and hence, it is called
tropopause.

• The stratosphere is found above the tropopause and extends up to a height of 50 km. One
important feature of the stratosphere is that it contains the ozone layer. This layer absorbs ultra-
violet radiation and this causes temperature to rise till it reaches stratopause. Temperature,
therefore first decreases in the troposphere, remains constant in the tropopause region and starts
increasing in the stratosphere till it reaches stratopause.
• The mesosphere lies above the stratosphere, which extends up to a height of 80 km. In this
layer, once again, temperature starts decreasing with the increase in altitude and reaches up to
minus 100°C at the height of 80 km. The upper limit of mesosphere is known as the mesopause.

• The ionosphere is located between 80 and 400 km above the mesopause. It contains
electrically charged particles known as ions, and hence, it is known as ionosphere. Temperature here
starts increasing with height. The uppermost layer of the atmosphere above the thermosphere is
also known as the exosphere.

Therefore, option (c) is the correct answer.

Subject: Geography | Structure of Atmosphere

Tags: Conceptual

Q27. With reference to ‘Commissions of Inquiries’, consider the following statements:

1. It is an inter-state body constituted under Article 263 of the Indian Constitution.

2. It has the powers of a civil court while trying a suit under the Code of Civil Procedure, 1908.

3. Only the Central Government has the power to establish such a commission.

Which of the statements given above is/are correct?

a) 1 and 2 only

b) 2 only

c) 3 only

d) 1, 2 and 3

Answer: b

Explanation:

• Parliament enacted the Commissions of Inquiry Act, 1952, which authorises the central and
state governments to appoint inquiry commissions to make inquiries in definite matters of public
importance. This is a central legislation enacted under the constitution, Schedule VII, List I and III. It
is not constituted under Article 263. So, statement 1 is not correct.

• Under The Commissions of Inquiry Act, 1952, a Commission set up by the government shall
have the powers of a civil court, while trying a suit under the Code of Civil Procedure, 1908. This
means that the Commission has powers to summon and enforce the attendance of any person from
any part of India and examine her on oath, and receive evidence, and it can order requisition of any
public record or copy from any court or office. So, statement 2 is correct.

• Commissions set up by the central government can make an inquiry into any matter
relatable to any of the entries in List I (Union List) or List II (State List) or List III (Concurrent List) in
the Seventh Schedule to the Constitution, while Commissions set up by state governments can look
into entries in List II or List III. If the central government set up the commission first, then states
cannot set up a parallel commission on the same subject matter without the approval of the Centre.
But if a state has appointed a Commission, then the Centre can appoint another on the same subject
if it is of the opinion that the scope of the inquiry should be extended to two or more states. So,
statement 3 is not correct.

Therefore, option (b) is the correct answer.

Relevance: The West Bengal government has set up a Commission of Inquiry into the alleged
surveillance of phones using the Pegasus spyware.

Subject: Polity | Bills and Acts

Tags: Factual

Q28. Consider the following statements:

1. Mosses reproduce by creating spores.

2. Majority of Lichen is made up of algae, which gives it its distinctive shape.

3. Blue-green algae are classified as bacteria.

Which of the statements given above is/are correct?

a) 1 and 2 only

b) 2 only

c) 1 and 3 only

d) 3 only

Answer: c

Explanation:

• Mosses: Mosses, also sometimes called bryophytes, are little flowerless plants that tend to
grow in tufts or mats. Rather than using flowers and seeds, mosses reproduce by creating spores
that are held in a little capsule. The lifecycle of moss can be divided into two stages, the
gametophyte stage and the sporophyte stage. So, statement 1 is correct.

• Lichens: Lichens form from a symbiotic relationship between two different organisms, a
fungus and an alga, meaning they are not plants at all. Each of these components provides critical
functions that help the individual survive. The fungus makes up most of the lichen, giving it its shape
and fruiting bodies while the algae provide the ability to photosynthesize. While algae normally need
a lot of water to survive, the fungus provides shelter from drying elements like heat and wind,
allowing lichens to exist even in hot dry climates. So, statement 2 is not correct.

• Blue-green algae: Although cyanobacteria are called blue-green algae, they are actually
bacteria, and are part of the bacteria kingdom, Monera. The "blue" in the common name refers to
the fact that they need to live in water, and "green algae" refers to their photosynthetic abilities, like
green algae. So, statement 3 is correct.

Therefore, option (c) is the correct answer.

Subject: Environment| Biodiversity & Conservation

Tags: Factual, Conceptual


Q29. With reference to the Scheduled Tribes and Other Traditional Forest Dwellers (Recognition of
Forest Rights) Act, 2006, which of the following are the functions of the Gram Sabha?

1. Initiating the process of determining the nature and extent of forest rights

2. Constituting Committees for the protection of wildlife, forest and biodiversity

3. Providing for the diversion of forest land for the facilities like schools and hospitals

Select the correct answer using the code given below.

a) 1 and 2 only

b) 2 and 3 only

c) 1 and 3 only

d) 1, 2 and 3

Answer: a

Explanation:

• Scheduled Tribes and Other Traditional Forest Dwellers (Recognition of Forest Rights) Act,
2006, has been enacted to recognize and vest the forest rights and occupation of forest land in
forest dwelling Scheduled Tribes and other traditional forest dwellers, who have been residing in
such forests for generations, but whose rights could not be recorded.

Provisions of the Scheduled Tribes and Other Traditional Forest Dwellers (Recognition of Forest
Rights) Act, 2006:

• According to the act, Gram Sabha means a village assembly which shall consist of all adult
members of a village and in case of States having no Panchayats, Padas, Tolas and other traditional
village institutions and elected village committees, with full and unrestricted participation of women

• Functions of Gram Sabha:

o Initiate the process of determining the nature and extent of forest rights, receive and hear
the claims relating thereto. So, statement 1 is correct.

o Prepare a list of claimants of forests rights and maintain a register containing such details of
claimants and their claims as the Central Government may by order determine;

o Pass a resolution on claims on forest rights after giving reasonable opportunity to interested
persons and authorities concerned and forward the same to the Sub-Divisional Level Committee;

o Constitute Committees for the protection of wildlife, forest and biodiversity, from amongst
its members, in order to carry out the provisions of the Act. So, statement 2 is correct.

• Notwithstanding anything contained in the Forest (Conservation) Act, 1980, the Central
Government shall provide for diversion of forest land for the facilities like dispensary or hospital,
schools, anganwadis, fair price shops etc. managed by the Government which involve felling of trees
not exceeding seventy-five trees per hectare. So, statement 3 is not correct.

Therefore, option (a) is the correct answer.


Relevance: Recently, Karnataka has ranked ninth in terms of approving applications filed by the
Scheduled Tribes (ST) and the Other Traditional Forest Dwellers (OTFD) under the Forest Rights Act
(FRA).

Subject: Environment | Acts and policies(env)

Tags: Factual

Q30. With reference to the Convention on Migratory Species (CMS), consider the following
statements:

1. It is an environmental treaty under the aegis of the United Nations.

2. All kind of agreements invoked under the convention are automatically legally binding.

3. The convention covers the species of migratory birds only.

Which of the statements given above is/are correct?

a) 1 only

b) 1 and 2 only

c) 3 only

d) 1, 2 and 3

Answer: a

Explanation:

Convention on Migratory Species (CMS):

• As an environmental treaty of the United Nations, CMS provides a global platform for the
conservation and sustainable use of migratory animals and their habitats. As the only global
convention specializing in the conservation of migratory species, their habitats and migration routes,
CMS complements and co-operates with a number of other international organizations, NGOs and
partners in the media as well as in the corporate sector. So, statement 1 is correct.

• Migratory species threatened with extinction are listed on Appendix I of the Convention.
CMS Parties strive towards strictly protecting these animals, conserving or restoring the places
where they live, mitigating obstacles to migration and controlling other factors that might endanger
them. Migratory species that need or would significantly benefit from international cooperation are
listed in Appendix II of the Convention. For this reason, the Convention encourages the Range States
to conclude global or regional agreements. In this respect, CMS acts as a framework Convention.

• The agreements may range from legally binding treaties (called Agreements) to less formal
instruments, such as Memoranda of Understanding, and can be adapted to the requirements of
particular regions. So, statement 2 is not correct.

• The conventions cover species of birds, fishes, mammals, reptiles, insects, etc. So, statement
3 is not correct.

• Nations like China, the USA, Russia, Canada and various nations of Southeast Asia are non-
parties to the convention. India became a party to the convention in November 1983.
Therefore, option (a) is the correct answer.

Relevance: A report, released by the United Nations Convention on the Conservation of Migratory
Species of Wild Animals (CMS), focused for the first time on the impacts of plastic pollution on
animals, birds, and aquatic beings in the Asia-Pacific region.

Subject: Environment | International Treaties

Tags: Factual

Q31. With reference to Peatlands, consider the following statements:

1. Fast rate of plant decomposition leads to the formation of peat.

2. These are the largest natural terrestrial carbon store.

3. These are found in Arctic tundra, coastal marshes and tropical swamp forests.

Which of the statements given above are correct?

a) 1 and 2 only

b) 2 and 3 only

c) 1 and 3 only

d) 1, 2 and 3

Answer: b

Explanation:

• Peatlands are a type of wetland that occur in almost every country and are known to cover
at least 3% of the global land surface. The term ‘peatland’ refers to the peat soil and the wetland
habitats growing on the surface. In peatlands, year-round water-logged conditions slow plant
decomposition to such an extent that dead plants accumulate to form peat. This stores the carbon
the plants absorbed from the atmosphere within peat soils, providing a net-cooling effect and
helping to mitigate the climate crisis. So, statement 1 is not correct.

• Peatlands are the largest natural terrestrial carbon store. They store more carbon than all
other vegetation types in the world combined. So, statement 2 is correct.

• Peatlands are found across the world: the arctic tundra, coastal marshes, tropical swamp
forests, mountainous fens and blanket bogs on subantarctic islands. So, statement 3 is correct.

• Damaged peatlands are a major source of greenhouse gas emissions, responsible for almost
5% of global anthropogenic CO2 emissions. Peatland restoration can reduce emissions significantly.

Therefore, option (b) is the correct answer.

Subject: Environment|Biodiversity & Conservation

Tags: Factual, Conceptual

Q32. With reference to the Marine Protected Areas (MPAs) in India, consider the following
statements:

1. MPA is notified under Wildlife Protection Act, 1972.

2. Protected Areas that fall entirely within the low tide line are included in the MPA

network.

3. Andaman and Nicobar Islands have the highest number of MPAs among all states and Union
Territories.

Which of the statements given above are correct?

a) 1 and 2 only

b) 2 and 3 only

c) 1 and 3 only

d) 1, 2 and 3

Answer: c

Explanation:

• To provide protection to the ecologically marine important areas, India initiated action
through the state governments to create a network of Marine Protected Areas under Wildlife
(Protection) Act, 1972. Gradually their number increased to cover critical and important marine
ecosystems. There are total of 31 major Marine Protected Areas in India covering coastal areas that
have been notified under Wildlife Protection Act, 1972.

• In India, Protected Areas (PAs) that fall entirely or partially within the swathe of 500 m from
the high tide line and the marine environment are considered to be in the MPA network. So,
statement 2 is not correct.

• The Wild Life Protection Act of India (1972) provides legal protection to many marine
animals. There are total of 31 Major Marine Protected Areas in India covering coastal areas that
have been notified under Wildlife Protection Act, 1972. So, statement 1 is correct.

• There are 105 PAs in the Andaman and Nicobar Islands (highest among all states and union
territories), and all are part of the MPA network of India. These MPAs cover about 60% of the
terrestrial area of the islands and protect more than 40% of the coastal habitat. So, statement 3 is
correct.

Therefore, option (c) is the correct answer.

Relevance: Recently the Minister of State (I/C) for M/o Earth Sciences in a written reply in Lok Sabha
mentioned about the several initiatives focusing on the conservation of coastal and marine
resources through implementation of law and continuous monitoring.

Subject: Environment| Protected areas

Tags: Factual
Q33. Consider the following statements:

1. Volcanic eruptions release iron into the water that can boost the growth of blue-green algae.

2. Volcanic emissions reaching the stratosphere can have a cooling effect on

global temperatures.

3. Erupting volcanoes can cause lightning.

4. Sulphur dioxide emissions from volcanoes are responsible for pollution in North India.

Which of the statements given above are correct?

a) 1 and 4 only

b) 2 and 3 only

c) 1, 2 and 3 only

d) 1, 2, 3 and 4

Answer: d

Explanation:

• A volcano is an opening or rupture in the earth’s surface that allows magma (hot liquid and
semi-liquid rock), volcanic ash and gases to escape. They are generally found where tectonic plates
come together or separate but they can also occur in the middle of plates due to volcanic hotspots.
A volcanic eruption is when lava and gas are released from a volcano—sometimes explosively.

• Falling ash from volcanoes could damage coral reefs. Volcanic Eruptions such as of Tonga
volcano (Undersea volcano, South Pacific) can release more iron into the water, which can boost the
growth of blue-green algae and sponges that further degrade reefs. So, statement 1 is correct.

• Volcanic lightning is a phenomenon that generally occurs at the early stages of a volcanic
eruption. It happens in two places: close to the ground in dense ash clouds, and high up near the
stratosphere in the plume of volcanic smoke. Volcanic emissions reaching the stratosphere can have
a cooling effect on global temperatures. So, statement 2 is correct.

• For volcanic lightning near the ground, research suggests the cause is the rubbing together
of individual ash particles, which builds up enough static electricity to generate a lightning bolt. So,
statement 3 is correct.

• Ninety-nine percent of the gas molecules emitted during a volcanic eruption are water vapor
(H2O), carbon dioxide (CO2), and sulfur dioxide (SO2). The remaining one percent is comprised of
small amounts of hydrogen sulfide, carbon monoxide, hydrogen chloride, hydrogen fluoride, and
other minor gas species. The sulphur dioxide (SO2) emissions from a volcanic eruption in the
Caribbean reached India April 16, 2021 sparking fear of increased pollution levels in the northern
parts of the country and acid rain. Sulphur dioxide reacts with water to form sulphuric acid which
can come down with rainfall. So, statement 4 is correct

Therefore, option (d) is the correct answer.

Relevance: Recently bolts of volcanic lightning were seen in the sky over eastern Sicily in Italy a
powerful eruption of Mount Etna.
Subject: Geography | Volcanoes

Tags: Factual, Conceptual, Analytical

Q34. With reference to Naturally Occurring Radioactive Materials (NORM), consider the following
statements:

1. They can be formed by the decay of thorium and uranium in earth’s crust.

2. They come to earth’s surface due to human activities only.

3. Technologically Enhanced NORM are those materials that are used as enhanced fuels in nuclear
power production.

Which of the statements given above is/are correct?

a) 1 only

b) 1 and 2 only

c) 3 only

d) 2 and 3 only

Answer: a

Explanation:

• Some Naturally occurring radioactive materials, often referred to using the short-form of
“NORM”, exists in the earth’s crust naturally. The decay of thorium and uranium produces many
different types of radioactive materials, called isotopes. Some examples of these isotopes are
Radium-226, Potassium-40, and Radon-222. These isotopes of NORM are generally found in the
earth’s crust in low concentrations. So, statement 1 is correct.

• NORM can come to the earth’s surface due to natural processes (radon gas moving through
cracks in rocks or dissolving and being transported by groundwater flows), or due to human activities
(mining, oil and gas extraction, etc.). Additionally, the human activities that bring NORM to the
surface may cause NORM to become more concentrated than its natural state. Because NORM is
radioactive, once it is brought to the surface or concentrated, the ionizing radiation that it emits can
now interact with humans. So, statement 2 is not correct.

• Technologically Enhanced Naturally Occurring Radioactive Material (TENORM) is defined as,


"Naturally occurring radioactive materials that have been concentrated or exposed to the accessible
environment as a result of human activities such as manufacturing, mineral extraction, or water
processing.” So, statement 3 is not correct.

Therefore, option (a) is the correct answer.

Subject: Science & technology|Radioactivity

Tags: Conceptual, Factual


Q35. When cutting an onion, which one of the following is the most likely cause of irritation in eyes?

a) Release of Formic acid

b) Release of Nitrogen oxide-based complex

c) Release of Lachrymatory-factor synthase

d) Release of Carbon Mono-oxide

Answer: c

Explanation:

• The lachrymatory factor is a chemical that irritates the eyes. When the onion is intact, a
group of compounds called cysteine sulfoxides are kept separate from an enzyme called alliinase.
But when you slice, dice or crush the onion, the barrier separating the compounds and enzyme is
broken. The two come together, setting off a reaction. The alliinase causes the cysteine sulfoxides to
become sulfenic acid.

• Sulfenic acids are not very stable, so they must change into something else. Option one is
that it can spontaneously condense, a reaction within itself, and become an organosulfur compound.
Organosulfur compounds are what give onions their strong smell and flavour. But option two for the
sulfenic acid is unique to onions and a couple of other alliums, or a genus of flowering plants that
produce vegetables such as onions, garlic, scallions and shallots. Another enzyme called
lachrymatory factor synthase, that's been hiding out in the cell comes into play and rearranges
sulfenic acid into the lachrymatory factor. The lachrymatory factor is a volatile liquid, meaning it
turns to vapour very quickly. That's how it reaches our eyes and irritates sensory nerves.

Therefore, option (c) is the correct answer.

Subject: Science & technology | Chemistry

Tags: Factual

Q36. In the context of Earth’s Moon, consider the following statements:

1. It is the largest moon orbiting in the solar system.

2. The Moon’s presence helps stabilize the earth’s wobble and moderate its climate.

3. Neither any country nor any space agency has explored the dark side of the moon till date.

Which of the statements given above is/are correct?

a) 1 and 2 only

b) 2 only

c) 3 only

d) 1, 2 and 3

Answer: b

Explanation:
• The Moon is Earth's only natural satellite. It goes around the Earth at a distance of about
385,000 kilometres. It is called "the Moon" because people didn't know other moons existed until
Galileo Galilei discovered four moons orbiting Jupiter in 1610.

• It is the fifth-largest of the 200+ moons orbiting planets in our solar system. Ganymede, a
satellite of Jupiter, has a radius of 1,635 miles (2,631 kilometres) and is the largest moon in our solar
system. It's bigger than Mercury and Pluto. So, statement 1 is not correct.

• Effect on Wobble and climate of Earth: The Moon’s presence helps stabilize our planet’s
wobble and moderate our climate. The Moon steadies the Earth as it spins on its axis, helping to give
us a stable climate. Without it, the Earth would wobble more erratically. The poles would move
markedly in relation to the Earth's orbit. Seasons, days and nights would all look very different. So,
statement 2 is correct.

• The atmosphere on Moon: The Moon has a very thin atmosphere called an exosphere. It
contains small amounts of a number of atoms and molecules including helium, argon, and possibly
neon, ammonia, methane and carbon dioxide. Moon's atmosphere is so thin, that atoms and
molecules almost never collide.

• Earth's Moon is the only place beyond Earth where humans have set foot. Humans first set
foot on the moon in July 1969.

• Farside of the moon: The Earth and Moon are tidally locked. Their rotations are so in sync
we only see one side of the Moon. Humans didn't see the lunar far side until a Soviet spacecraft flew
past in 1959. In 2019, for the first time in history, a space mission touched down on the far side of
the moon. China's Chang'e-4 probe landed in the oldest and deepest basin on the moon's surface.
Making the mission a milestone for both China and space exploration as a whole. So, statement 3 is
not correct.

Therefore, option (b) is the correct answer.

Subject: Science & technology |Space

Tags: Conceptual, Analytical, Factual

Q37. Shooting stars that we see in the sky are

a) Comets passing the revolutionary axis of the Earth

b) Meteoric particles intercepting the Earth

c) Pulsars emitted by rotating neutron stars

d) Particles of explosion in a distant star

Answer: b

Explanation:

• A shooting star is a common name for a meteor, or a space rock that collides with Earth's
atmosphere. Meteors that are called shooting stars appear as a flash of light.

• This light is the visible trail of gasses in Earth's upper atmosphere that is heated to
incandescence by the high-speed passage of a meteoroid, or meteoric particle, intercepting the
Earth. Quite simply, a shooting star is a piece of space rock or dust that briefly becomes visible
when it begins to burn up in our planet's atmosphere. Much of this material comes from the
asteroid belt between Mars and Jupiter.

Therefore, option (b) is the correct answer.

Subject: Science & technology |Space

Tags: Conceptual

Q38. Adenine, guanine, thymine and cytosine are related to

a) naturally occurring antibodies

b) bindings protein found on RNA

c) digestive enzymes

d) bases found in DNA

Answer: d

Explanation:

Adenine, guanine, thymine, and cytosine:

• A base pair is two chemical bases bonded to one another forming a "rung of the DNA
ladder." The Deoxyribonucleic acid (DNA) molecule consists of two strands that wind around each
other like a twisted ladder. Each strand has a backbone made of alternating sugar (deoxyribose) and
phosphate groups.

• Attached to each sugar is one of four bases--adenine (A), cytosine (C), guanine (G), or
thymine (T). The two strands are held together by hydrogen bonds between the bases, with adenine
forming a base pair with thymine, and cytosine forming a base pair with guanine.

Therefore, option (d) is the correct answer.

Subject: Science & technology | Human Body

Tags: Conceptual

Q39. Which one of the following statements is not correct?

a) In Type 1 diabetes, the pancreas makes very little or no insulin.

b) Type 2 diabetes most often develops in people over the age of 45

c) Type 2 diabetes is less common than Type 1 diabetes.

d) Both Type 1 and Type 2 diabetes can be diagnosed by a simple blood test.

Answer: c

Explanation:
• Type 1 diabetes: If a person has type 1 diabetes, her/his pancreas doesn’t make insulin or
makes very little insulin. Insulin is a hormone made by your pancreas that acts like a key to let blood
sugar into the cells in your body for use as energy.

• Type 1 diabetes is thought to be caused by an autoimmune reaction (the body attacks itself
by mistake). This reaction destroys the cells in the pancreas that make insulin, called beta cells.

• Type 2 diabetes: If a person has type 2 diabetes, cells don’t respond normally to insulin; this
is called insulin resistance. Type 2 diabetes most often develops in people over age 45, but more and
more children, teens, and young adults are also developing it.

• Type 1 diabetes is less common than type 2, about 5-10% of people with diabetes have type
1.

• A simple blood test will let a person know if he/she has type 1 or type 2 diabetes. A fasting
blood sugar level from 100 to 125 mg/dL (5.6 to 6.9 mmol/L) is considered prediabetes. If it's 126
mg/dL (7 mmol/L) or higher on two separate tests, you have diabetes.

Therefore, option (c) is the answer.

Relevance: A new study has revealed that over 55 per cent of newly diagnosed type 2 diabetes
mellitus (T2DM) patients in India have low HDL-C (High Density Lipid – Cholesterol) values, indicating
that they are at higher risk of developing some form of cardiovascular disease in their lifetime.

Subject: Science & technology |Health

Tags: Conceptual

Q40. Which of the following are generally used as preservatives in Pickles?

1. Salt

2. Sugar

3. Acetic Acid

4. Fumaric Acid

5. Sulphur dioxide

Select the correct answer using the code given below.

a) 1 and 2 only

b) 3 and 4 only

c) 1, 2, 3 and 4 only

d) 1, 2, 3, 4 and 5

Answer: c

Explanation:

• On a most general level, pickles are foods soaked in solutions that help prevent spoilage.
There are two basic categories of pickles. The first type includes pickles preserved in vinegar, a
strong acid in which few bacteria can survive. The other category includes pickles soaked in brine to
encourage fermentation, growth of “good” bacteria that make a food less vulnerable to “bad”
spoilage causing bacteria. Pickling foods does much more than simply preserve them. It can also
change their taste and texture in several interesting ways.

• Preservatives are added to pickles so as to prevent or slow down the growth of micro-
organisms, such as moulds, yeasts and bacteria in food. Preservatives can inhibit, retard or arrest the
process of fermentation, acidification or other deterioration of pickles.

• As per The Food Safety and Standards (Food Products Standards and Food Additives)
Regulations, 2011 has classified preservatives in to classes. These preservatives are used in pickling:

o Class – I Preservatives: Salt, Sugar, Vinegar or Acetic Acid, Spices v. Edible Vegetable Oils. So,
points 1, 2 and 3 are correct.

o Class – II Preservatives: Benzoic Acid - 250 ppm (Maximum), Sulphur dioxide - 100 ppm
(Maximum). So, point 5 is correct.

• Fumaric acid: It is an important specialty chemical with wide industrial applications ranging
from its use as feedstock for the synthesis of polymeric resins to acidulant in foods and
pharmaceuticals. So, point 4 is not correct.

Therefore, option (c) is the correct answer.

Subject: Science & technology | Chemistry

Tags: Factual

Q41. Which of the present-day religious practices were also a part of religious practice of the Indus
Valley Civilization?

1. Swastika symbol

2. Phallus Worship

3. Worship of Pashupati

4. Fire Altars

5. Worship of Peepal tree

6. Ritual Bath

Select the correct answer using the code given below.

a) 2, 4 and 6 only

b) 3, 4 and 5 only

c) 1, 2, 3 and 6 only

d) 1, 2, 3, 4, 5 and 6

Answer: d

Explanation:
• From the seals, terracotta figurines and copper tablets we get an idea on the religious life of
the Harappans.

• Swastika motif is depicted on terracotta, shell, faience, and metal tablets. It may have been
associated with a protective function or auspiciousness. So, point 1 is correct.

• Aspect of the fertility-related beliefs of the Harappans was the worship of male and female
creative energy in the form of stone icons of lingas (Phallus) and yonis. A number of such stones
were identified. So, point 2 is correct.

• It is suggested that the Harappans also worshipped a male god represented on a steatite
seal discovered at Mohenjodaro, usually referred to as the Pashupati seal. It shows resemblance to
the Shiva of later Hindu mythology, who is also known as Mahayogi (the great yogi) and Pashupati
(lord of the animals). So, point 3 is correct.

• The most striking evidence suggesting ritualistic practices comes from the ‘fire altars’ found
on the citadel mound at Kalibangan containing cattle bones and antlers. This suggests the practice of
animal sacrifice. ‘Fire altars’ have also been reported at Banawali, Lothal, Amri, Nageshwar, and
Vagad in Gujarat and at Rakhigarhi in Haryana. So, point 4 is correct.

• The Harappan seals, sealings, amulets, and copper tablets depict a number of trees, plants,
and animals, some of which may have had cultic significance. The pipal tree appears often and may
have been venerated. Sometimes, there is a figure peering out from between its branches, possibly a
tree-spirit. A seal found at Mohenjo-daro shows a row of seven figures with long braids standing in
front of a pipal tree which has a horned figure standing in it. So, point 5 is correct.

• Great Bath at Mohenjodaro is suggested to serve as a ritual bathing site. The floor of the
Bath was made of burnt bricks. Water was drawn from a large well in an adjacent room, and an
outlet from one corner of the bath led to a drain. Immediately to the north of the Great Bath is a
large building consisting of eight small rooms with common bathing platforms. So, point 6 is correct.

Therefore, option (d) is the correct answer.

Subject: History| Indus Valley Civilization

Tags: Factual, Analytical

Q42. With reference to the Dhamma policy of Ashoka, consider the following statements:

1. Good conduct and social responsibilities were part of this policy.

2. Slavery was banned in the empire for promotion of respectful behavior towards all.

3. Royal pleasure tours were replaced by dhamma tours.

Which of the statements given above is/are correct?

a) 1 only

b) 2 only

c) 2 and 3 only

d) 1 and 3 only
Answer: d

Explanation:

• Ashoka adopted the policy of Dhamma in the 13th year after his consecration. The edicts
were written primarily to explain to the people throughout the empire the principles of Dhamma. He
also appointed a special cadre of Dhamma mahamatras to propagate dhamma. It was a way of life, a
code of conduct and a set of principles to be adopted and practised by the people at large. His
principles of Dhamma were clearly stated in his Edicts.

• Good conduct and social responsibilities were part of dhamma. Rock edict 9 begins with a
criticism of ceremonies performed by people, especially women, on occasions such as illness,
marriage, birth, and setting forth on journeys. Such rituals are described as producing uncertain and
meagre results. Ashoka contrasts these with the ceremony of dhamma, which is bound to yield
results in this world (i.e. life) and the next. So, statement 1 is correct.

• The ceremony of dhamma is described as consisting of proper courtesy to slaves and


servants, respectful behaviour towards elders, restraint in one’s dealings with all living beings, and
liberality to shramanas and Brahmanas. Thus, Ashoka did not ban slavery. So, statement 2 is not
correct.

• The chief disseminator of the dhamma message was Ashoka himself. In major rock edict 8,
he states that earlier kings used to go on pleasure tours consisting of hunts and other pastimes. Ten
years after his abhisheka, he made a pilgrimage to Bodh Gaya. Thenceforth, the royal pleasure tours
(vihara-yatas) were replaced by dhamma tours (dhamma-yatas). The latter involved visiting
Brahmanas and shramanas and giving them gifts, visiting aged folk and distributing gold to them,
meeting people of the countryside, instructing them in dhamma, and questioning them about
dhamma. Ashoka asserts that he derived more pleasure from these dhamma tours than from
anything else. So, statement 3 is correct.

Therefore, option (d) is the correct answer.

Subject: History|Maurya Dynasty

Tags: Conceptual, Analytical

Q43. With reference to Mathura School of Art, consider the following statements:

1. It is completely indigenous and shows no trace of foreign influence.

2. Its theme included Buddhism, Jainism and Hinduism.

3. Most of the sculptures were made of lime plaster.

Which of the statement given above are correct?

a) 1 and 2 only

b) 2 and 3 only

c) 1 and 3 only

d) 1, 2 and 3
Answer: a

Explanation:

• Mathura was one of the pre-eminent cities of north India. It was the southern capital of the
Kushanas and an important centre of crafts and trade, religious activity, and artistic production.

• Mathura sculptures share iconographic similarities with those of the northwest, but the style
is very different. It is completely indigenous and shows no trace of foreign influence. The Mathura
style can be seen as a further development of the traditions of sculpture of sites such as Besnagar,
Sanchi, and Bharhut. So, statement 1 is correct.

• The themes were varied, including yakshas, yakshis, nagas, nagis, Buddhas and bodhisattvas,
Jaina tirthankaras, and Hindu deities. So, statement 2 is correct.

• The early centuries CE marked an explosion in the number and variety of Vaishnava images
produced in the Mathura area. It was during this period, Mathura became the premier centre of the
dissemination of Vaishnava sculptural art. The sculptures included kinship triads depicting Vasudeva
Krishna, his brother Baladeva, and their sister Ekanamsha. There are many independent images of
Vasudeva Krishna, but also some of Vishnu (four-armed), Vishnu on garuda, and in anthropomorphic
boar form.

• The sculptors of Mathura used red sandstone quarried at Sikri. So, statement 3 is not
correct.

Therefore, option (a) is the correct answer.

Subject: History | Art and culture

Tags: Conceptual, Analytical

Q44. Consider the following statements:

1. The Vikram era is believed to have been established by King Vikramaditya of Ujjain to
commemorate his victory over the Saka rulers.

2. The national calendar of India is based on the Saka Era, with Chaitra as its first month.

Which of the statements given above is/are correct?

a) 1 only

b) 2 only

c) Both 1 and 2

d) Neither 1 nor 2

Answer: c

Explanation:

• Saka Samvat: This calendar form was initiated by King Shalivahan in 78 CE. It was also known
as the Saka era as it is to this tribe that Shalivahan belonged. Historians do have a confusion of
whether Shalivahan was Saka himself or conquered Sakas.
• Vikram Samvat: The Vikram era started in 56 BCE and is in force in almost all of India except
the region of Bengal. This era, as historians believe, is said to have been established by King
Vikramaditya of Ujjain to commemorate his victory over the Saka rulers. So, statement 1 is correct.

• The national calendar of India is based on the Saka Calendar which has been adopted as the
official civil calendar besides the Gregorian calendar. The Saka Calendar is based on luni-solar
reckoning of time. The calendar consists of 365 days and 12 months like the normal Gregorian
calendar. Chaitra is the first month of the year beginning on March 22 which is the day after the
Spring Equinox. During leap years, the starting day of Chaitra corresponds with March 21. So,
statement 2 is correct.

Therefore, option (c) is the correct answer.

Subject: History | Miscellaneous(hist)

Tags: Factual

Q45. With reference to ancient centres of learning, consider the following pairs:

SI. No. Centres Built/Patronised

by

1. Odantapuri : Gopala I

2. Nalanda : Harshvardhan

3. Vikramshila : Dharampala

4. Somapura : Kanishka

Which of the pairs given above are correctly matched?

a) 1 and 2 only

b) 2, 3 and 4 only

c) 1 and 4 only

d) 1, 2 and 3 only

Answer: d

Explanation:

• Odantapuri: It is located in Bihar and was made under the patronage of Pala Dynasty King
Gopala I. It was a Buddhist mahavihara which was destroyed by Bakhtiyar Khilji. So, pair 1 is correctly
matched.

• Nalanda: The most renowned university of South Asia. It is not clear as to who established it;
it was in existence during the Gupta period. It gained prominence under Harshavardhana’s reign and
Pala kings. All three Buddhist doctrines were taught here, however, it was a major site for Mahayana
Buddhist teachings. So, pair 2 is correctly matched.

• Vikramshila: It is located in present-day Bhagalpur district of Bihar. It was established by King


Dharampala of Pala Dynasty, primarily as a Buddhist learning centre. The scholars were invited by
kings outside India to spread Buddhist teachings. The Vajrayana sect of Buddhism flourished here
and Tantric teachings were taught. So, pair 3 is correctly matched.

• Somapura: This Buddhist monastery was built in the style of a university during the Pala
period around the 8th century by Dharmapala. It was a renowned intellectual centre until the 12th
century. Geographically located to the north-west of Bangladesh in the district of Naogaon close to
the village of Paharpur. So, pair 4 is not correctly matched.

Therefore, option (d) is the correct answer.

Subject: History| Miscellaneous(hist)

Tags: Factual

Q46. With reference to Indian heritage, which of the following belong to the category of rock-cut
architecture?

1. Masroor Temples

2. Ajanta caves

3. Shore temple of Mahabalipuram

4. Kailashnath temple of Kanchipuram

5. Pancha Rathas of Mahabalipuram

Select the correct answer using the code given below.

a) 1, 2 and 5 only

b) 2, 3 and 5 only

c) 3 and 4 only

d) 2 and 5 only

Answer: a

Explanation:

• The Masroor Temples, also referred to as Rock-cut Temples at Masroor, is an early 8th-
century complex of rock-cut Hindu temples in the Kangra Valley of Beas River in Himachal Pradesh,
India. The temples face northeast, towards the Dhauladhar range of the Himalayas. They are a
version of North Indian Nagara architecture style, dedicated to Shiva, Vishnu, Devi and Saura
traditions of Hinduism, with its surviving iconography likely inspired by a monotheistic framework.
So, point 1 is correct.

• Ajanta is a series of rock-cut caves in the Sahyadri ranges on Waghora River near
Aurangabad in Maharashtra. There are a total of 29 caves there of which 25 were used as Viharas or
residential caves while 4 were used as Chaitya or prayer halls. The caves were developed in the
period between 200 to 650 CE. The Ajanta caves were inscribed by the Buddhist monks, under the
patronage of the Vakataka kings, Harishena being a prominent one. So, point 2 is correct.
• Under Rajasimha, development of real structural temples started in place of rock-cut
temples in the Pallava empire. Shore Temple at Mahabalipuram, Kailashnath Temple at Kanchipuram
both are structural temples. So, points 3 and 4 are not correct.

• Ratha Temples or Pancha Ratha (also known as Pandava Rathas), are the earliest rock cut
temples in India, comprising of Dharmaraja Ratha, Bhima Ratha, Arjuna Ratha, Nakula and Sahadeva
Ratha, and Draupadi Ratha dated around 7th century CE. So, point 5 is correct.

Therefore, option (a) is the correct answer.

Subject: History | Temple Architecture

Tags: Factual

Q47. Consider the following statements:

1. Marco Polo visited the Kakatiya kingdom during the reign of Queen Rudramadevi.

2. Domingo Paes reported advanced irrigation technology of the Vijayanagar Empire.

3. Willian Hawkins visited the Mughal court during the reign of Aurangzeb.

4. Afanasy Nikitin described his travels to India in a short travelogue, ‘A Journey Beyond the Three
Seas’.

Which of the statements given above are correct?

a) 1 and 4 only

b) 2 and 3 only

c) 1 and 3 only

d) 1, 2 and 4 only

Answer: d

Explanation:

• Marco Polo was an Italian merchant, adventurer and writer who visited India towards the
end of the 13th century. Polo documented his travels in ‘The Travels of Marco Polo’. He visited the
Pandyan Kingdom and left a vivid description of the land and its people. Marco Polo visited the
Kakatiya kingdom in around 1289 CE during the reign of Queen Rudramadevi. It can be found from
his works that a woman named Rudramadevi had inherited her father’s throne and ruled the
dynasty as a kind and benevolent ruler. So, statement 1 is correct.

• Domingo Paes was a Portuguese merchant, writer and explorer who visited India between
1520–1522 CE and gave the most detailed account of the ancient city of Hampi under the reign of
King Krishnadeva Raya. He reported the following features about Vijayanagara empire:

o Advanced irrigation technology that allowed the peasant to produce high yielding crops at
very low prices. So, statement 2 is correct.

o A wide variety of cultures were shown in crops and vegetation.

o He described a busy market of precious stones.


o The city was prospering and its size was comparable to Rome, with abundant vegetation,
aqueducts and artificial lakes.

• William Hawkings (1608-1611 CE) was a representative of the English East India Company
and an ambassador of the British King James-I. He came to India in 1608 and travelled to Agra in the
court of Mughal Emperor Jahangir to negotiate for the establishment of a factory at Surat. So,
statement 3 is not correct.

• Afanasy Nikitin (1446-1472 CE) was a Russian merchant from Tver, a city not far from
Moscow. He described his travels in a short travelogue known as A Journey Beyond the Three Seas.
He visited Bahamani Empire under reign of Muhammad Shah III. He documented a battle between
the Bahmani Empire and Vijayanagar. So, statement 4 is correct.

Therefore, option (d) is the correct answer.

Subject: History|Travellers

Tags: Factual

Q48. Consider the following pairs:

SI. No. Temples Situated on the

bank of

1. Omkareshwar : Narmada

2. Brihadeshwara : Kaveri

3. Kamakhya : Kameng

4. Trimbakeshwar : Krishna

Which of the pairs given above is/are correctly matched?

a) 1 and 2 only

b) 2 only

c) 3 only

d) 1, 2, 3 and 4

Answer: a

Explanation:

• Omkareshwar is a famous pilgrimage centre that includes one of the 12 Jyotirlinga shrines of
Shiva. It is located on the banks of river Narmada.It is located in Khandwa in Indian state Madhya
Pradesh. So, pair 1 is correctly matched.

• Brihadeshwara Temple (also known as RajaRajeswara temple) is a Hindu temple dedicated


to Shiva located in Thanjavur in the Indian state of Tamil Nadu. It is one of the largest temples in
India and is an example of Dravidian architecture during the Chola period. Built by emperor Raja Raja
Chola I and completed in 1010 CE, the temple is part of the UNESCO World Heritage Site known as
the “Great Living Chola Temples”. It is located on the Southern banks of river Kaveri. So, pair 2 is
correctly matched.
• The majestic Kamakhya Temple is one of the oldest temples in Guwahati, Assam dating back
to the 8th century. One of the Shakti Peeths of India, it is situated atop the Nilachal Hills in the
western part of Guwahati in Assam. The mighty Brahmaputra river flows through the northern banks
of the temple. Ambubachi festival is held every year during Monsoon to celebrate the annual
menstruation of Goddess Kamakhya. So, pair 3 is not correctly matched.

• Trimbakeshwar town is an ancient Hindu Pilgrim centre located at the source of the
Godavari River, the longest river in peninsular India. Trimbakeshwar is the abode of one of the
twelve Jyotirlingas. The extraordinary feature of the Jyotirlinga located here is that the Linga in the
temple is in the form of a three faced embodying Tridev, Lord Brahma, Lord Vishnu and Lord Shiva.
The present Trimbakeshwar temple was constructed by third Peshwa Balaji Bajirao (1740-1760) on
the site of an old temple. So, pair 4 is not correctly matched.

Therefore, option (a) is the correct answer.

Subject: History | Temples

Tags: Factual

Q49. With reference to Kathakali, consider the following statements:

1. It is a pure dance form performed in Tamil Nadu.

2. Its characters are broadly divided into satvika, rajasika and tamasika types.

3. It is the only classical dance that uses Hindustani music.

4. Kathakali is exclusively performed by women.

Which of the statements given above is/are correct?

a) 1 only

b) 2 only

c) 2 and 4 only

d) 1, 2 and 3 only

Answer: b

Explanation:

• Kathakali, as a dance form popular today, is considered to be of comparatively recent origin.


However, it is an art which has evolved from many social and religious theatrical forms which existed
in the southern region in ancient times. Chakiarkoothu, Koodiyattam, Krishnattam and Ramanattam
are few of the ritual performing arts of Kerala which have had a direct influence on Kathakali in its
form and technique. Legend has it that the refusal of the Zamorin of Calicut to send his Krishnattam
troupe to Travancore, so enraged the Raja of Kottarakkara, that he was inspired to compose the
Ramanattam.

• Kathakali is a classical dance form from Kerala. It is a blend of dance, music and acting and
dramatises stories, which are mostly adapted from the Indian epics. It is a stylised art form, the four
aspects of abhinaya - angika, aharya,vachika, satvika and the nritta, nritya and natya aspects are
combined perfectly. So, statement 1 is not correct.

• Kathakali dance is chiefly interpretative. The characters in a Kathakali performance are


broadly divided into satvika, rajasika and tamasika types. Satvika characters are noble, heroic,
generous and refined. In pacha, green colour dominates and kirita (headgear) is worn by all. Krishna
and Rama wear special crowns decorated with peacock feathers. The noble characters like Indra,
Arjun and the Devas are some of the pacha characters. So, statement 2 is correct.

• The dancers wear makeup typical of their character, known as vesham. The pacha vesham
with its predominant green colour is used to portray noble male characters like kings and divine
beings. Kathi characters are arrogant and evil but have a streak of valour in them. The characters of
the thadi (beard) category are the chuvanna thadi, (red beard), vellathadi (white beard) and the
karutha thadi (black beard). Kari vesham is used for demonic characters, portraying the most
gruesome figures on the Kathakali stage. Minukku vesham symbolises gentleness and high spiritual
qualities (like saints), which are in sharp contrast to the preceding four classes.

• Kathakali music follows the traditional sopana sangeet of Kerala. It is said to be the ritual
singing of the Ashtapadis on the flight of steps leading to the sanctum sanctorum. Now, Kathakali
music also uses Carnatic ragas. The orchestra, which is also used in other traditional performing arts
of Kerala, normally comprises the Chenda, Maddalam, Chengila, Ilathalam, Idakka and Shankhu.
Kathak (not Kathakali) is the only form of classical dance wedded to Hindustani or the North Indian
music. So, statement 3 is not correct.

• Kathakali has always been considered a male art form. It originated as part of the bhakti
movement and is believed to have been performed by soldiers of the kingdom, as the plays were
written by a prince himself. Since they were well-trained in the martial art of kalari, troupes that
followed in their footsteps all across the land also implemented rigorous exercises as part of the
training process. In recent times, women also started participating in Kathakali. So, statement 4 is
not correct.

Therefore, option (b) is the correct answer.

Subject: History| Dances

Tags: Factual

Q50. The text – ‘Kalak Acharya Katha’, belongs to

a) Shaivism

b) Buddhism

c) Jainism

d) Vaishnavism

Answer: c

Explanation:

• The Kalpasutra and the Kala Acharya-Katha, the two very popular Jain texts were repeatedly
written and illustrated with paintings commissioned by the Jain merchants and bankers.
• Kalpasutra is a Prakrit work composed by Bhadrabahu in the 5th century BC. This canonical
work narrates the early stages of development of Jainism.

• Kalak Acharya Katha tells the story of a great Jain muni of the Shwetambar school called
Kalak. Kalaka, a Jain teacher who uses his magical powers to help the Saka (Scythian). In return they
help him defeat the evil king of Ujjain Gardhabilla, who had kidnapped his sister. This story is very
often found as an appendix to the Kalpasutra.

Therefore, option (c) is the correct answer.

Subject: History | Religions: Buddhism Jainism and Sikhism(hist)

Tags: Factual

Q51. With reference to Vedic Period, consider the following pairs:

SI. No. Sacrifice and

Rituals Purpose

1. Rajasuya : Attainment of power

and prosperity

2. Agnihotra : Purifying the

atmosphere

3. Vajapeya : Royal consecration

ceremony

4. Ashvamedha : Political paramountcy

Which of the pairs given above are correctly matched?

a) 1 and 4 only

b) 1 and 3 only

c) 2 and 4 only

d) 2 and 3 only

Answer: c

Explanation:

• Larger kingdoms were formed during the later Vedic period. Many jana or tribes were
amalgamated to form janapadas or rashtras in the later Vedic period. Hence the royal power had
increased along with the increase in the size of the kingdom. The king performed various rituals and
sacrifices to strengthen his position.

• The rajasuya was the royal consecration ceremony. Apart from a number of agrarian fertility
rites, it included a ritual cattle raid, in which the rajan raided the cattle of his kinsmen, and also a
game of dice, which the king won. At a larger, symbolic level, in the rajasuya, the king was presented
as standing in the centre of the cyclical processes of regeneration of the universe. So, pair 1 is not
correctly matched.
• The Agnihotra was a simple domestic sacrifice, to be performed daily by the head of a dvija
household. Also called daiva yajna, havana or homa, it is the Vedic way of purifying the atmosphere
by fumigating clarified butter, grains, odoriferous substances, dried cow-dung cakes, herbal powder
and sticks of specified trees, along with the chanting of mantras. So, pair 2 is correctly matched.

• The vajapeya sacrifice was connected with the attainment of power and prosperity, and also
contained a number of fertility rites. It included a ritualchariot race in which the rajan raced against
his kinsmen and defeated them. So, pair 3 is not correctly matched.

• The ashvamedha was a horse sacrifice associated with claims to political paramountcy and
incorporated several fertility rites as well. So, pair 4 is correctly matched.

Therefore, option (c) is the correct answer.

Subject: History| Vedic era

Tags: Factual

Q52. Who among the following authored “The Arctic Home in the Vedas”, a debate over the

origins and the original homeland of the Aryans in the Arctic?

a) Madan Mohan Malviya

b) Aurobindo Ghosh

c) Bal Gangadhar Tilak

d) Veer Savarkar

Answer: c

Explanation:

• Bal Gangadhar Tilak’s campaign against the British colonial rule was anchored on reclaiming
Indian heritage and culture, some of which were reflected in his 1892 paper, “The Orion, or
Researches into the Antiquity of the Vedas”, which drew on the vast knowledge of the Hindu Vedas
and Avesta, the sacred scripture of Zoroastrianism. In sum, it was a search into the antiquity of the
Aryan-Vedic culture, challenging the ancientness of the Bible and establishing the primacy of the
Vedic literature.

• Tilak’s articulation of the Vedic culture was further elaborated in his 1904 work, ‘’The Arctic
Home of the Vedas’’ that brought in an Indian consciousness to human social evolution as a
response to the physical environment. While updating his estimate of the Vedic antiquity, Tilak came
to the conclusion that the advancement of geological science, astronomy and archaeology proved
that the climate conditions at the pole during the interglacial period was conducive for human
habitation establishing, thereby, his long-held belief that the ancestors of India’s ancient Vedic
civilisation lived in the Arctic region.

• Sri Aurobindo, the renowned Vedic scholar, in his work, The Secret of the Vedas, appreciates
Tilak’s interpretation noting that by examining the “Vedic Dawn, the figure of the Vedic cows, and
the astronomical data of the hymns”, a strong probability of the Aryan race “descending from the
Arctic regions in the glacial period” emerges.
Therefore, option (c) is the correct answer.

Subject: History | Important Personalities

Tags: Factual

Q53. In the context of Colonial India, Stafford Cripps, Pethick-Lawrence and A.V. Alexander were the
members of

a) First Education Committee set up in India.

b) Indian Statutory Commission

c) Boundary Commission set up under Cyril Radcliffe

d) Cabinet Mission Plan

Answer: d

Explanation:

• On February 19, 1946, Lord Pethick Lawrence made an important announcement in the
House of Lords that a special mission of Cabinet ministers consisting of Noble Lord himself, the
President of the Board of Trade, Sir Stafford Cripps and the First Lord of the at Admiralty, Mr AV
Alexander would go to India to seek in association with the Viceroy an agreement with Indian
leaders of the principles and procedures relating to the constitutional issue.

• The mission recommended that the Constitution of India should take the following basic
forms:

o There should be Union of India, embracing both British India and the States, which should
deal with the following subjects: foreign affairs, defence and communications, and should have the
power to raise the finances required for the above subjects

o The Union should have an Executive and Legislature constituted from British India and the
States’ representatives. Any question facing a major communal issue in the legislature should
require for its decision, a majority of the representatives present and voting, of each of the two
major communities as well as a majority of all members present and voting.

o Provinces would enjoy full autonomy, all subjects other than the Union subjects and all
residue powers should vest in the provinces.

o Moreover, “Provinces should be free to form groups with Executive and legislature, and each
group could determine the provincial subjects to be taken in common”.

o The six Hindu majority provinces i.e Madras, Bombay, Central province, United province,
Bihar and Orissa would form group A. The Muslim majority provinces in the north west — the
Punjab, the NWFP and Sind would form group B. Bengal and Assam would form group C. The chief
commissioners provinces— Delhi, Ajmer-Marwar, Coorg would join group A and Balochistan would
join group B.

Therefore, option (d) is the correct answer.

Subject: Modern History | Indian freedom struggle

Tags: Factual
Q54. With reference to Women freedom fighters in India, consider the following statements:

1. Matangini Hazra participated in Civil Disobedience Movement and was arrested for violating the
Salt Act.

2. Annie Besant wrote the poem 'The gift of India’.

3. Sarla Devi Chaudharani was the founder of the first women’s organisation in India, the Bharat
Stree Mahamandal.

Which of the statements given above is/are correct?

a) 1 only

b) 1 and 3 only

c) 2 and 3 only

d) 1, 2 and 3

Answer: b

Explanation:

• Matangini Hazra was the Gandhian leader who participated in the Civil Disobedience
movement. She also participated in the salt satyagraha, manufactured salt at Alinan salt Centre and
was arrested for violating the salt act. She also participated in the Chowkidar Tax Bandha movement.
So, statement 1 is correct.

• Sarojini Naidu joined the Indian national movement in the wake of partition of Bengal in
1905. She came in contact with Gopal Krishna Gokhale, Rabindranath Tagore and Mahatma Gandhi.
In 1925, she presided over the annual session of Indian National Congress at Kanpur. She wrote the
poem ‘the Gift of India’. So, statement 2 is not correct.

• Sarala Devi Chaudhurani was the founder of the first women’s organisation in India, the
Bharat Stree Mahamandal in Allahabad in 1910. One of the primary goals of the organisation was to
promote female education, which at that time was not well developed. The organisation opened
several offices in Lahore, Delhi, Karachi, Amritsar, Hyderabad, Kanpur, Hazaribagh, Midnapore and
Kolkata to improve the situation of women all over India. So, statement 3 is correct.

Therefore, option (b) is the correct answer.

Subject : Modern History | Indian freedom struggle

Tags: Factual

Q55. Who among the following leaders were actively involved in the Non-Cooperation Movement of
1920?

1. Bal Gangadhar Tilak

2. C.R Das

3. Vallabhbhai Patel

Select the correct answer using the code given below.

a) 1 and 2 only
b) 2 and 3 only

c) 1 and 3 only

d) 1, 2 and 3

Answer: b

Explanation:

• The Non-Cooperation Movement was formally launched on 1 August 1920. That day was
also marked by the death of Lokmanya Tilak, which witnessed strikes and processions to mourn the
passing way of this great national leader. So, point 1 is not correct.

• A special session of the Congress was held in Calcutta in September 1920 to finally
deliberate and decide the issue of non-cooperation. It gave its assent to non-cooperation, despite
some opposition by those interested in Council entry. By the time of the regular Congress Session in
December 1920 held at Nagpur, the programme of non-cooperation was accepted without
opposition. It involved the surrender of government titles and honorary positions, boycott of
government schools and colleges, law courts, and foreign cloth. It could also be extended to include
resignation from government service and nonpayment of government taxes. Moreover, it was
decided to set up national schools and colleges, establish and strengthen the panchayats for
settlement of disputes, promotion of hand spinning and weaving, condemnation and renunciation of
untouchability, maintenance of communal amity and strict observance of non-violence. Thus, for the
first time, an open extra-constitutional programme of mass mobilisation was started by the
Congress.

• Although the boycott of the law courts was not as successful, many leading lawyers of the
country like C.R. Das, Motilal Nehru, Saifuddin Kitchlew, C. Rajagopalachari, M.R. Jayakar,
Vallabhbhai Patel, Asaf Ali and T. Prakasham quit their practice. The boycott of foreign cloth became
a very successful programme. So, point 2 and point 3 are correct.

Therefore, option (b) is the correct answer.

Subject: Modern History | Indian Freedom Struggle

Tags: Factual

Q56. With reference to the Gandhian concept of Sarvodaya, consider the following statements:

1. It provides for the welfare and greatest happiness of the greatest number of people.

2. The main features of Sarvodaya are trusteeship and economic equality.

3. The idea of Sarvodaya was majorly influenced by John Ruskin’s work – Unto This Last.

Which of the statements given above are correct?

a) 1 and 2 only

b) 2 and 3 only

c) 1 and 3 only

d) 1, 2 and 3
Answer: b

Explanation:

• Sarvodaya is a Sanskrit word derived from two words, namely, sarva and udaya. Sarva means
‘all’ which includes every kind of living beings. Udaya means ‘rise,’ ‘uplift,’ ‘prosperity,’ ‘affluence,’
etc. So Sarvodaya literally means ‘the welfare of all’ or ‘the upliftment of all. By the concept
Sarvodaya Gandhi really means universal uplift or the welfare of all men and women and not just the
welfare, or greatest happiness, of the greatest number, which is rather a utilitarian concept.
Sarvodaya is the name Gandhi gives to the new society embracing the betterment of the entire
humankind and the world at large. It is a society directed towards the integral welfare of all living
beings based on age and on the old spiritual and moral values of Indian and those found elsewhere.
Gandhi was not the supporter of utilitarianism which aims at the greatest happiness to the greatest
number. Sarvodaya aims at the integral liberation of every individual. It also stands for the well-
being of the whole individual, body and soul. So, statement 1 is not correct.

• The main features of Sarvodaya are trusteeship, economic equality and networking. So,
statement 2 is correct.

o The theory of trusteeship, elimination of exploitation in every shape or form; a classless


society which offers no privileges by the birth or wealth or talent; mutual cooperation being the
driving force of motivation and behaviour; and above all, securing the welfare of all without any
distinction of race, religion, sex, political affiliation: these may be said to be the highlights of the
Sarvodaya society envisioned by the Mahatma.

o According to Gandhi economic equality is the master key to non-violent transformationary


independence. Working for economic equality means abolishing the eternal conflict between capital
and labour. A non-violent system of government is clearly and impossibly so long as the wide gulf
between the rich and the hungry millions persists. The most disadvantaged sections of the
community being the economically, the physically and the mentally poor, it is obvious that no
egalitarian society could evolve without the poor taking part in the evolutionary process.

o During his lifetime, Gandhi never allowed Sarvodaya organisations to be formally instituted.
His thinking was that once you served people, as ‘sevakas,’ within the Sarvodaya philosophy there
was no need for institutionalization. But since his demise Sarvodaya organisations have been
established world-wide genuinely helping people towards building new societies from the
‘grassroots’ up. At the operational field level, the most successful Sarvodaya programmes have most
often been those which, apart from being self-created and self-managed, have employed practical
hands-on-learning-by-doing methods, including a lot of income generation, skills training and
conscientisation; rather than the theoretical ‘talk-shop’ or workshop approach.

• The socio-political philosophy of Gandhi may be termed Sarvodaya. It may, however, be


noted that Gandhi was not the author of the idea of Sarvodaya. He borrowed the idea as well as the
term from Ruskin. Ruskin himself had borrowed it from the Bible. Gandhi first used the word
Sarvodaya in a booklet which he published in his native Gujarati after he read John Ruskin. The
essay, Unto This Last, was based on a parable from the Gospel according to Matthew 20:1-16
concerning the owner of a plantation and his hiring of labourers to work in his vineyard. In his
parable Jesus make the master of the vineyard declare: “I will give unto this last even as I give unto
thee.” The emphasis of Ruskin’s essay, as interpreted by Gandhi, is certainly that ideal society is
none in which there is concern for the welfare of all. This is the foundation for his Sarvodaya
movement. Along with Unto This Last Tolstoy’s Bread Labour also influenced Gandhi. However, the
credit of Gandhi lies in the fact that he devoted his entire life to the creation of a new society, a
society based on the principles of Sarvodaya. So, statement 3 is correct.

Therefore, option (b) is the correct answer.

Subject: History | Gandhi – Advent and Achievement

Tags: Factual

Q57. With reference to Chennakesava Temple located in Belur, consider the following statements:

1. It was built during the reign of the Hoysala dynasty.

2. It is made of marble with very detailed carvings.

3. It is situated on the banks of river Krishna.

4. Though a Vaishnava temple, it houses images from Jainism and Buddhism.

Which of the statements given above are correct?

a) 1 and 2 only

b) 2 and 3 only

c) 1 and 4 only

d) 2, 3 and 4 only

Answer: c

Explanation:

• The Chennakeshava Temple, also referred to as Keshava, or Vijayanarayana Temple of Belur,


Karnataka is one of the bigger and more renowned Hoysala temples. The temple was built in the
early 12th century by the Hoysala ruler, Vishnuvardhana, when the town of Belur was the site for the
capital of the Hoysala kingdom. So, statement 1 is correct.

• Built with soapstone, the Chennakesava temple features a very detailed finish built around a
typical Hoysala style blueprint. So, statement 2 is not correct.

• The temple stands on a platform or Jagati and looks like a huge casket. The pillars of the
temple exhibit some of the best details and finishing of sculpture and art work in the entire complex.
The Narasimha pillar is one of the most popular of these temple pillars.

• The Chennakeshava Temple is built on the banks of the Yagachi River in Belur, also known as
Velapura. So, statement 3 is not correct.

• Though a Vaishnava temple, it includes themes from Shaivism and Shaktism, as well as
images from Jainism and Buddhism. The Chennakeshava temple is a testimony to the artistic,
cultural and theological perspectives in 12th century South India and the Hoysala era, which was
tolerant towards all beliefs. So, statement 4 is correct.

Therefore, option (c) is the correct answer.

Relevance: The Hoysala temples at Belur, Halebidu and Somnathpur are India’s official nominations
for inclusion in the UNESCO World Heritage Sites.
Subject: History|Temple Architecture

Tags: Factual, Analytical, Current Affairs

Q58. Which of the following Harappan cities specialised in shell working?

1. Chanhudaro

2. Balakot

3. Amri

4. Kot Diji

5. Dholavira

Select the correct answer using the code given below.

a) 2 and 3 only

b) 1, 2 and 5 only

c) 1 and 4 only

d) 1, 2, 4 and 5 only

Answer: b

Explanation:

• Making shell objects was an important Harappan craft. Beads, bracelets, Bangles and
decorative inlay work of shell have been discovered from many cities. Chanhudaro and Balakot were
important centres of shell work.

• Further evidence of site specialisation comes from Gujarat. An intensive surface survey and
excavations at Nageshwar (in Jamnagar district) have shown that this site was exclusively devoted to
shell-working and specialised in making bangles.

• Evidence of shell working also comes from Kuntasi, Dholavira, Rangpur, Lothal, Nagwada,
and Bagasra. This craft was clearly very important in the Gujarat region of the Harappan culture
zone.

Therefore, option (b) is the correct answer.

Relevance: Researchers will study shells from Indus Valley Civilization for signs of climate change.

Subject: History | Indus Valley Civilization

Tags: Factual

Q59. ‘Botulinum, Furocoumarins and Lectins’, often mentioned in the news, are

a) natural toxins

b) growth hormones

c) antiviral compounds
d) animal enzymes

Answer: a

Explanation:

Botulinum:

• Botulinum toxin, one of themost poisonous biological substances known, is a neurotoxin


produced by the bacterium Clostridium botulinum. Botulinum toxins block nerve functions and can
lead to respiratory and muscular paralysis. Foodborne botulism, caused by the consumption of
improperly processed food, is a rare but potentially fatal disease if not diagnosed rapidly and treated
with antitoxin. Homemade canned, preserved or fermented foodstuffs are a common source of
foodborne botulism and their preparation requires extra caution.

Furocoumarins:

• These toxins are present in many plants such as parsnips (closely related to carrots and
parsley), celery roots, and citrus plants (lemon, lime, grapefruit, bergamot) and some medicinal
plants. Furocoumarins are stress toxins and are released in response to stress, such as physical
damage to the plant. Some of these toxins can cause gastrointestinal problems in susceptible
people. Furocoumarins are phototoxic, they can cause severe skin reactions under sunlight (UVA
exposure).

Lectins:

• Lectins Many types of beans contain toxins called lectins, and kidney beans have the highest
concentrations, especially red kidney beans. As few as 4 or 5 raw beans can cause severe
stomachache, vomiting and diarrhoea. Lectins are destroyed when the dried beans are soaked for at
least 12 hours and then boiled vigorously for at least 10 minutes in water. Tinned kidney beans have
already had this process applied and so can be used without further treatment.

Therefore, option (a) is the correct answer.

Relevance: Wildlife authorities in Assam recently said they had found the carcasses of nearly 100
endangered vultures. Vultures have an extremely corrosive stomach acid that allows them to
consume rotting animal corpses. These scavenged leftovers are often infected with anthrax,
botulinum toxins and rabies that would otherwise kill other animals.

Subject: Environment |Pollution

Tags: Factual

Q60. With reference to the Deepor Beel, consider the following statements:

1. It is the only Ramsar site in Assam.

2. It is a permanent freshwater lake.

3. It drains into the Brahmaputra River.

Which of the statements given above are correct?

a) 1 and 2 only
b) 2 and 3 only

c) 1 and 3 only

d) 1, 2 and 3

Answer: d

Explanation:

Deepor Beel:

• Deepor Beel Bird Sanctuary, southwest of Guwahati city, is located in Kamrup (metropolitan)
district of Assam, on the southern bank of the Brahmaputra river. It was designated as a Ramsar site
on 19-08-2002. It is the only Ramsar Site in Assam. So, statement 1 is correct.

• The sanctuary is a permanent freshwater lake with abundant aquatic vegetation. Deepor is
one of the largest of many such lakes or ‘beels’ in lower Assam. So, statement 2 is correct.

• The main sources of water for this lake are the Basistha and Kalamoni rivers, and local
monsoon run-off between May and September. The Beel drains into the Brahmaputra river, located
about 5 km away. So, statement 3 is correct.

o An endorheic basin is a drainage basin that normally retains water and allows no outflow to
other external bodies of water, such as rivers or oceans, but drainage converges instead into lakes or
swamps, permanent or seasonal, that equilibrate through evaporation.

• The beel is a staging site on migratory flyways and some of the largest concentrations of
aquatic birds in Assam can be seen, especially in winter. Some globally threatened birds are
supported, including Spot-billed Pelican (Pelicanus philippensis), Lesser and Greater Adjutant Stork
(Leptoptilos javanicus and dubius), and Baer's Pochard (Aythya baeri). Despite Deepor Beel’s
proximity to the sprawling Guwahati city and nearby settlements, the Asian Elephant regularly visits
the beel. In the nearby Rani reserve forest and other forested areas, mammals such as Hoolock
Gibbon, Assamese Macaque, Rhesus Macaque, Capped Langur, Slow Loris, Leopard, Jungle Cat,
Leopard Cat, three species of Civets, Sambar, Barking Deer, Chinese Pangolin have been recorded.

Therefore, option (d) is the correct answer.

Relevance: Recent reports have suggested that Deepor Beel, a perennial freshwater lake and the
only Ramsar site in Assam, has shrunk around 35 per cent in size since 1991.

Subject: Environment |Protected areas

Tags: Factual

Q61. With reference to Neanderthals, consider the following statements:

1. They were generally shorter and had more robust skeletons than modern humans.

2. Their Brain size was larger than the average modern human brain.

3. They became extinct during lower Palaeolithic period.

4. Denisovans were a group of Neanderthals from Siberia.


Which of the statements given above are correct?

a) 1, 2 and 3 only

b) 1 and 2 only

c) 2 and 4 only

d) 3 and 4 only

Answer: b

Explanation:

Neanderthal:

• Neanderthals (Homo neanderthalensis) are members of a group of archaic humans who


emerged at least 200,000 years ago during the Pleistocene Epoch (about 2.6 million to 11,700 years
ago). Neanderthals lived during the Ice Age. They often took shelter from the ice, snow and
otherwise unpleasant weather in Eurasia's plentiful limestone caves. Many of their fossils have been
found in caves, leading to the popular idea of them as "cave men." Like other humans, Neanderthals
originated in Africa but migrated to Eurasia long before other humans did. Neanderthals lived across
Eurasia, as far north and west as the Britain, through part of the Middle East, to Uzbekistan.

• Body size and shape: Neanderthals were generally shorter and had more robust skeletons
and muscular bodies than modern humans. brain size was larger than the average modern human
brain and averaged 1500 cubic centimetres. This is expected, as Neanderthals were generally heavier
and more muscular than modern humans. So, statements 1 and 2 are correct.

• Extinction of Neanderthals: Scientists broadly agree that the Neanderthals died out about
40,000 years ago, after a wave of modern humans migrated out of Africa about 20,000 years earlier.
The Lower Paleolithic period traditionally ranges from 2.6 million years ago to 200,000 years ago. So,
statement 3 is not correct.

Denisovans:

• While placed in the genus Homo, the Denisovans still have no agreed taxonomic name. They
are named after the Denisova Cave, Siberia, Russia, where the first fossils were found and identified.
Evidence suggests that Neanderthals, Denisovans, and modern humans are all descended from or
share a common ancestor with Homo heidelbergensis. They were not a group of neanderthals but
rather distinct from them. So, statement 4 is not correct.

• In many ways, Denisovans resembled Neanderthals but in some traits they resembled
humans and in others they were unique. Denisovans are thought to have been based in Siberia and
eastern Asia. Scientists have found evidence that the Denisovans lived at high altitudes in Tibet,
passing on a gene that helps modern people cope at similar elevations.

Therefore, option (b) is the correct answer.

Subject: Environment |Evolution

Tags: Factual, Conceptual


Q62. Which of the following features are common among “Platypus and Echidna”?

1. They are egg-laying mammals

2. They are amphibious mammals

3. They are non-venomous

4. They can be found in Australia

Select the correct answer using the code given below.

a) 1 and 2 only

b) 1, 2 and 3 only

c) 1 and 4 only

d) 3 and 4 only

Answer: c

Explanation:

Echidnas:

• They are also called spiny anteaters. Echidnas, together with the platypus, are the world's
only monotremes, or egg-laying mammals. There are two species of echidnas: the long-beaked
echidna, which is confined to the highlands of New Guinea; and the short-beaked echidna is
common throughout most of temperate Australia and lowland New Guinea. So, statement 1 is
correct.

• Echidnas are found throughout New Guinea and mainland Australia, as well as Tasmania,
King Island, Flinders Island and Kangaroo Island. They are Australia's most widespread native
mammal, being found in almost all habitats, from snow-covered mountains to deserts. They are also
common in urban areas, although their camouflage can make them very difficult to see. Echidnas
are usually found among rocks, in hollow logs and in holes among tree roots. During rainy or windy
weather they often burrow into the soil or shelter under bushes and tussocks of grass. So, statement
2 is not correct.

Platypus:

• Also called duckbill, the platypus is a duck-billed, beaver-tailed, otter-footed, egg-laying


aquatic mammal native to Australia. While the platypus generally inhabits freshwater rivers,
wetlands, and billabongs Down Under, it is also known to venture into brackish estuaries. The
platypus is a bottom-feeder that uses its beaver-like tail to steer and its webbed feet to propel itself
through the water while hunting for insects, shellfish, and worms. So, statement 4 is correct.

• Male platypuses have a spur on the inner side of each ankle that is connected to a venom
gland located over the thighs. The spurs can be wielded in defense, and the venom is potent enough
to kill small animals and cause intense pain in humans if the spur penetrates the skin. So, statement
3 is not correct.

Therefore, option (c) is the correct answer.

Subject: Environment |Biodiversity & Conservation


Tags: Factual, Conceptual

Q63. With reference to the flow of energy in an ecosystem, consider the following statements:

1. Ecological efficiency is the efficiency at which energy is transferred from one trophic level to
another.

2. More than half of the energy is transferred from one trophic level to another.

3. Assimilation efficiency is the efficiency by which animals convert the food they ingest into energy.

4. Herbivores generally have higher assimilation efficiencies than carnivores.

Which of the statements given above are correct?

a) 1 and 3 only

b) 1, 2 and 3 only

c) 1, 3 and 4 only

d) 2 and 4 only

Answer: a

Explanation:

• Ecological efficiency (also called Lindman’s efficiency) is the efficiency at which energy is
transferred from one trophic level to another. On average it is estimated that there is only a 10
percent transfer of energy. Lindman in 1942 defined these ecological efficiencies for the 1st time
and proposed a 10% rule e.g. if autotrophs produce 100 cal, herbivores will be able to store 10 cal.
and carnivores 1 cal. However, there may be slight variations in different ecosystems and ecological
efficiencies may range from 5 to 35%. So, statement 1 is correct and statement 2 is not correct.

• Energy is lost in several ways as it flows along these pathways of consumption. Most plant
tissue is uneaten by herbivores, and this stored energy is therefore lost to the plant-herbivore-
carnivore food chain. In terrestrial communities less than 10 percent of plant tissue is actually
consumed by herbivores. The rest falls into the detritus pathway, although the detritivores consume
only some of this decaying tissue. Oil and coal deposits are major repositories of this unused plant
energy and have accumulated over long periods of geologic time.

• The efficiency by which animals convert the food they ingest into energy for growth and
reproduction is called assimilation efficiency. So, statement 3 is correct.

• Herbivores assimilate between 15 and 80 per cent of the plant material they ingest,
depending on their physiology and the part of the plant that they eat. For example, herbivores that
eat seeds and young vegetation high in energy have the highest assimilation efficiencies, those that
eat older leaves have intermediate efficiencies, and those that feed on decaying wood have very low
efficiencies. Carnivores generally have higher assimilation efficiencies than herbivores, often
between 60 and 90 percent, because their food is more easily digested. So, statement 4 is not
correct.

Therefore, option (a) is the correct answer.

Subject: Environment |Energy flow in ecosystem


Tags: Conceptual, Factual, Analytical

Q64. Which one of the following is the reason for corals being generally absent on the west coast of
continents?

a) Less salinity in water on the east coasts of continents

b) Cold ocean currents and upwelling of cold water on the west coasts of continents

c) High ocean pollution on the west coasts of continents

d) Presence of dry climate and subsidence of air in the west coasts of continents

Answer: b

Explanation:

• Coral reefs begin to form when free-swimming coral larvae attach to submerged rocks or
other hard surfaces along the edges of islands or continents. As the corals grow and expand, reefs
take on one of three major characteristic structures — fringing, barrier or atoll. All three reef types
— fringing, barrier and atoll — share similarities in their biogeographic profiles. Bottom topography,
depth, wave and current strength, light, temperature, and suspended sediments all act to create
characteristic horizontal and vertical zones of corals, algae and other species.

• Reef-building corals are restricted in their geographic distribution by their physiology. For
instance, reef-building corals cannot tolerate water temperatures below 18° Celsius (C). Many grow
optimally in water temperatures between 23° and 29° C, but some can tolerate temperatures as high
as 40° C for short periods. Most also require very saline (salty) water ranging from 32 to 42 parts per
thousand, which must also be clear so that a maximum amount of light penetrates it. The corals’
requirement for high light also explains why most reef-building species are restricted to the euphotic
zone, the region in the ocean where light penetrates to a depth of approximately 70 meters.

• Corals are generally absent on the western coast of continents because the western coast
has cold currents which reduce water temperature below 18 degree sometimes. Hence corals
generally remain absent.

• In India, corals found in Gulf of Kutch, Gulf of Mannar, Palk Bay, Andaman & Nicobar and
Lakshadweep Island. The coastal line has some coral deposits, and beautiful beaches. These islands
receive convectional rainfall and have an equatorial type of vegetation. The islands of the Arabian
sea include Lakshadweep and Minicoy. These are scattered between 8°N-12°N and 71°E -74°E
longitude. These islands are located at a distance of 280 km-480 km off the Kerala coast. The entire
island group is built of coral deposits.

Therefore, option (b) is the correct answer.

Subject: Environment | Biodiversity & Conservation

Tags: Factual

Q65. Which one of the following statements best describes the term ‘Environmental Gradient?

a) It is the change in the composition of species from one ecosystem to another.

b) It is the change in a biome with the change in the latitude.


c) It can be described by changes in environmental characteristics such as elevation, soil acidity and
soil composition.

d) It is the change in both biotic and abiotic factors with a change in depth in an aquatic ecosystem.

Answer: c

Explanation:

Environmental gradients:

• Environmental gradients relate to variations in site characteristics and may be described by


changes in elevation, changes in site index, or variations in other factors such as soil acidity, soil
composition, soil fertility, soil moisture, exposure, edges and interior areas, and even land-use
history.

• It results in different ecological characteristics with changes in gradient. For example, some
tree species are found in greater abundance on sites with a particular aspect (e.g., southern or
southeastern); with a higher or lower slope position (e.g., ridge versus valley); that is more
productive or have greater soil moisture availability, or within a specific elevational range. White oak
(Quercus alba) in the eastern United States, for example, generally prefers south-facing slopes, high
slope positions, and xeric (i.e., dry) soils. Trees may respond differently to the resources available in
different gradients at different life stages. For example, oak trees at an intermediate life stage (e.g.,
saplings) may be more sensitive to environmental gradients than oak trees at a seedling or mature
life stage.

Therefore, option (c) is the correct answer.

Subject: Environment |Ecology

Tags: Conceptual

Q66. With reference to Generics and Biosimilar drugs, consider the following statements:

1. In both, the active ingredients are different from the originator.

2. Generics generally have smaller molecules compared to structurally complex molecules of


biosimilars.

3. Compared to generics, biosimilars have significantly higher research and development costs and
risks.

Which of the statements given above are correct?

a) 1 and 2 only

b) 2 and 3 only

c) 1 and 3 only

d) 1, 2 and 3

Answer: b

Explanation:
• Generic drugs and biosimilars serve similar purposes: They are both unbranded versions of
existing medications typically provided at a lower cost. Generic medicines are chemically derived
drugs designed to be equivalent to an existing, authorized originator. Only the inactive ingredients of
a generic are allowed to differ from the originator. Because the chemical entities at the core of these
drugs are exact copies of the originators, their performance in patients is predictable, and these
chemicals are relatively uncomplicated to synthesize. By contrast, biosimilars are developed to be
similar to biological medicines that have already been approved and whose patent has expired. The
active ingredient of the biosimilar and the originator are functionally the same, but there could be
minor differences because of production methods and the complexity of the molecules involved. So,
statement 1 is not correct.

• Biosimilars are inherently different from generics due to their molecular size and structure,
and the complexity and cost of their development. Generics generally have smaller molecules that
areeasy to characterize. Biosimilar generally have structurally complex molecules, that are more
difficult to characterize. So, statement 2 is correct.

• Biosimilars also have significantly higher research and development costs and risks and are
more complex to manufacture than small-molecule generics. So, statement 3 is correct.

Therefore, option (b) is the correct answer.

Subject: Science & technology | Health

Tags: Conceptual, Factual

Q67. Consider the following statements:

1. Facial recognition software reads the facial features like distance between the eyes and the
distance from forehead to chin.

2. Facial recognition software generates facial signature that is based on a mathematical formula.

3. 3D facial recognition scans can be performed in good lighting conditions only and can be used for
digital payments.

Which of the statements given above are correct?

a) 1 and 2 only

b) 2 and 3 only

c) 1 and 3 only

d) 1, 2 and 3

Answer: a

Explanation:

• We can trace the history of facial recognition to the 1960s. That’s when mathematician and
computer scientist Woodrow Wilson Bledsoe first developed a system of measurements that could
be used to put photos of faces in different classifications. Because of this work, Bledsoe is known as
the unofficial father of facial recognition technology.

• Facial recognition system varies with technologies, but here are the basic steps:
o Step 1: A picture of your face is captured from a photo or video. Your face might appear
alone or in a crowd. Your image may show you looking straight ahead or nearly in profile.

o Step 2: Facial recognition software reads the geometry of your face. Key factors include the
distance between your eyes and the distance from forehead to chin. The result: your facial
signature. So, statement 1 is correct.

o Step 3: Your facial signature, a mathematical formula, is compared to a database of known


faces. So, statement 2 is correct.

o Step 4: A determination is made. Your faceprint may match that of an image in a facial
recognition system database.

• 3D facial recognition method: It involves using sensors to capture the shape of the face with
more precision. Unlike traditional facial recognition methods, the accuracy of 3D facial recognition is
not affected by lighting, and scans can even be done in the dark. Another advantage of 3D facial
recognition is that it can recognize a target from multiple angles, rather than just a straight-on
profile. So, statement 3 is not correct.

• Digital payments: The Moscow metro has recently rolled out what authorities have lauded
as the world’s first mass-scale facial recognition payment system. The cashless, cardless and
phoneless system, named Face Pay, launched at more than 240 stations across the Russian capital.

Therefore, option (a) is the correct answer.

Relevance: The Moscow metro has recently rolled out FacePay

Subject: Science & technology | Facial recognition

Tags: Conceptual, Factual

Q68. With reference to Internet Protocol (IP), consider the following statements:

1. An IP address is a unique address that identifies a device on the internet or a local network.

2. IP address is allocated by the International Telecommunication Union (ITU).

3. IPv4 is based on 32-bit addressing whereas IPv6 is based on 128-bit addressing.

4. IPv6 provides simpler and shorter IP addresses than IPv4.

Which of the statements given above are correct?

a) 1, 2 and 3 only

b) 1 and 3 only

c) 1, 2 and 4 only

d) 3 and 4 only

Answer: b

Explanation:
• An IP address is a unique address that identifies a device on the internet or a local network.
IP stands for "Internet Protocol," which is the set of rules governing the format of data sent via the
internet or local network. In essence, IP addresses are the identifier that allows information to be
sent between devices on a network: they contain location information and make devices accessible
for communication. So, statement 1 is correct.

• Allocation of IP addresses: IP addresses are not random. They are mathematically produced
and allocated by the Internet Assigned Numbers Authority (IANA), a division of the Internet
Corporation for Assigned Names and Numbers (ICANN). ICANN is a non-profit organization that was
established in the United States in 1998 to help maintain the security of the internet and allow it to
be usable by all. So, statement 2 is not correct.

IPv4 and IPv6:

• IPv4 is based on 32-bit addressing, limiting it to a total of 4.3 billion addresses. IPv6 is based
on 128-bit addressing and can support 340 undecillion addresses. Having more addresses has grown
in importance with the expansion of smart devices and connectivity. So, statement 3 is correct.

• IPv4 addresses contain a series of four numbers, ranging from 0 (except the first one) to 255,
each separated from the next by a period, such as 5.62.42.77. IPv6 addresses are represented as
eight groups of four hexadecimal digits, with the groups separated by colons. A typical IPv6 address
might look like this: 2620:0aba2:0d01:2042:0100:8c4d:d370:72b4. So, statement 4 is not correct.

Therefore, option (b) is the correct answer.

Subject: Science & technology | Computer Technology

Tags: Conceptual, Factual

Q69. Consider the following statements:

1. Both Ballistic and Cruise Missiles are powered by jet engines throughout their trajectory.

2. Intercontinental Ballistic Missiles (ICBMs) have a range greater than 5500 km.

3. Ballistic Missiles cannot be launched from air due to their large size and weight.

4. Both Cruise and Ballistic Missiles can carry nuclear warheads.

Which of the statements given above are not correct?

a) 1 and 2 only

b) 2 and 3 only

c) 3 and 4 only

d) 1 and 3 only

Answer: d

Explanation:

Ballistic missiles:
• Ballistic missiles are powered initially by a rocket or series of rockets in stages, but then
follow an unpowered trajectory that arches upwards before descending to reach its intended target.
Ballistic missiles can carry either nuclear or conventional warheads. So, statement 1 is not correct.

• There are four general classifications of ballistic missiles based on their range, or the
maximum distance the missile can travel:

o Short-range: less than 1,000 kilometres (approximately 620 miles), also known as “tactical”
ballistic missiles.

o Medium-range: between 1,000 and 3,000 kilometres (approximately 620-1,860 miles), also
known as “theater” ballistic missiles.

o Intermediate-range: between 3,000 and 5,500 kilometres (approximately 1,860-3,410 miles)

o Long-range: more than 5,500 kilometres (approximately 3,410 miles), also known as
intercontinental or strategic ballistic missiles. Intercontinental ballistic missiles (ICBMs) can fly much
further than the minimum range. So, statement 2 is correct.

Air-launched ballistic missiles (ALBM):

• In 2018, Russia and China both tested an uncommon type of missile that flew a ballistic
trajectory but could be launched from an aircraft. ALBMs are not new, the United States, Russia,
Israel, and now China all have considerable experience with ALBM technology. While the United
States developed an ALBM option in the 1960s to supplement its strategic nuclear forces. So,
statement 3 is not correct.

Cruise missiles:

• Cruise missiles are unmanned vehicles that are propelled by jet engines, much like an
airplane. They can be launched from ground, air, or sea platforms. Capable of carrying either a
nuclear or a conventional warhead, the cruise missile was designed to have a very low radar cross
section and to hug the ground while traveling at a relatively slow speed to its target. Cruise missiles
are self-guided and use multiple methods to accurately deliver their payload, including terrain
mapping, global positioning systems (GPS) and inertial guidance, which uses motion sensors and
gyroscopes to keep the missile on a pre-programmed flight path. So, statement 4 is correct.

Therefore, option (d) is the answer.

Subject: Science & technology | Defence technology

Tags: Conceptual, Analytical

Q70. Which of the following is/are the applications of Doppler Effect?

1. Detection of planets outside of the solar system

2. Detection of water in the atmosphere

3. Measuring the distance of a body from Earth

4. Measuring the speed of a car

Select the correct answer using the code given below.

a) 1 and 3 only
b) 2, 3 and 4 only

c) 1, 2 and 4 only

d) 1, 2, 3 and 4

Answer: c

Explanation:

• Doppler effect, the apparent difference between the frequency at which sound or light
waves leave a source and that at which they reach an observer, caused by the relative motion of the
observer and the wave source.

Applications of the Doppler effect:

• Astronomers rely on the Doppler effect to detect planets outside of our solar system or
exoplanets. To date, 442 of the 473 known exoplanets have been detected using the Doppler effect.
So, point 1 is correct.

• Doppler shifts occur as the star orbits around its own center of mass and moves toward or
away from Earth. How a planet’s Doppler shift changes over time can also shed light on the planet’s
orbital period, the shape of its orbit and its minimum possible mass.

• By sending radar beams into the atmosphere and studying the changes in the wavelengths
of the beams that come back, meteorologists use the Doppler effect to detect water in the
atmosphere. So, point 2 is correct.

• The Doppler phenomenon is also used in healthcare with echocardiograms that send
ultrasound beams through a body to measure changes in blood flow to make sure that a heart valve
is working properly or to diagnose vascular diseases.

• Police also rely on the Doppler effect when they use a radar gun to bounce radio beams off
of a car; the change in frequency between the directed and reflected beams provides a measure of a
car’s speed. So, point 4 is correct.

Measuring distance of a body from Earth:

• Parallax is the difference in direction of a celestial object as seen by an observer from two
widely separated points. The two positions of the observer and the position of the object form a
triangle; if the baseline between the two observing points is known and the direction of the object
as seen from each has been measured, the apex angle (the parallax) and the distance of the object
from the observer can be found simply.

• The measurement of parallax is used directly to find the distance of the body from Earth
(geocentric parallax) and from the Sun (heliocentric parallax). So, point 3 is not correct.

Therefore, option (c) is the correct answer.

Relevance:

Subject: Science & technology|Physics

Tags: Conceptual, Factual


Q71. With reference to the agrarian structure in medieval India, consider the following statements:

1. Richer Peasants or peasant proprietors were termed khud-kasht.

2. Pahi-kasht were tenant cultivators who migrated from other villages.

3. Those cultivators who did not have lands of their own were known as muzarian.

Which of the statements given above is/are correct?

a) 1 only

b) 1 and 2 only

c) 3 only

d) 1, 2 and 3

Answer: d

Explanation:

• The village population comprised different sections and categories of people, each with
different functions and status. The superior section of the village society was composed of
zamindars, muqaddams, chaudhari, qanungo (rural aristocracy). They owed their status partly to
hereditary superior right in land and partly to their position in the apparatus of revenue
administration.

• Peasants were not a homogeneous group. The rich peasants often formed part of the rural
elite. They were commonly termed as khud-kasht, kalantaran, or paltis and halmir in Persian
documents. They possessed their own granary, well, house, and ploughs. They used to get their land
cultivated with the help of hired labour in addition to their family labour. So, statement 1 is correct.

• Pahis (their counterpart in the Deccan was upari) were not the resident cultivator but they
were peasants cultivating the lands in villages other than their own. Usually they were the migrants
from the neighbouring villages/parganas to the villages either deserted or where cultivable land was
available. They were generally assessed at concessional rates (1/3 of the produce). The village patel
(village headmen) normally played an important role in bringing these new asamis (pahis). State
encouraged the pahis to settle in new villages. In such cases ploughs, oxen, manure and money were
provided by the state. So, statement 2 is correct.

• Those cultivators who did not have lands of their own were known as muzarian. The
muzarian were the tenants who used to cultivate the land of superior castes/ landholders. They also
served as state sponsored tenants. In that case they were asked to cultivate surplus lands or
abandoned lands. In the village there were also share-croppers. They were referred to in Rajasthani
documents as sanjhedars. So, statement 3 is correct.

Therefore, option (d) is the correct answer.

Subject: History |Medieval India

Tags: Factual
Q72. With reference to the Congress Seva Dal, consider the following statements:

1. It was established at the Belgaum Session of the Indian National Congress in 1924.

2. Subhash Chandra Bose was its first President and had Jawaharlal Nehru and Umabai Kundapur as
its members.

Which of the statements given above is/are correct?

a) 1 only

b) 2 only

c) Both 1 and 2

d) Neither 1 nor 2

Answer: d

Explanation:

• Congress Seva Dal is the grassroot frontal organisation of Indian National Congress (INC).
The organisation has a chapter in all the states and union territories of the India and is present in
over 700+ districts today.

• On January 1, 1924 the Congress Seva Dal was established as the Hindustan Seva Mandal
with Jawaharlal Nehru as its first president. So, statement 2 is not correct.

• According to the resolution at Kakinada Session of the INC, the Dal was to work under the
supervision of the Congress party's working committee. Umabai Kundapur was the founding
president of the women's wing of the Dal. So, statement 1 is not correct.

Therefore, option (d) is the correct answer.

Subject: Modern History | Indian freedom struggle

Tags: Factual

Q73. With reference to Government of India Act, 1919, consider the following statements:

1. It introduced an elected majority in both houses.

2. It established a responsible government at the Centre.

3. It gave the power to the members to move resolutions and motions for adjournment of the
House.

Which of the statements given above is/are correct?

a) 1 only

b) 2 and 3 only

c) 1 and 3 only

d) 1, 2 and 3

Answer: c
Explanation:

• The Government of India Act, 1919, set up a bicameral legislature at the Centre in place of
the Imperial Council consisting of one house. The two houses now were to be the Council of State
and the Central Legislative Assembly. The Council of State which was to be the Upper House, was to
consist of 60 members, 26 of whom were to be nominated by the Governor General and 34 were to
be elected, introduced there by an elected majority. The Legislative Assembly which was to
constitute the Lower House was to consist of 145 members, of whom 41 were to be nominated and
104 elected. So, statement 1 is correct.

• The Act of 1919, did not introduce any responsible government at the Centre. No vote of no
confidence by the legislature to turn out the members of the Executive Council of the Governor
General and as such they were irremovable. So, statement 2 is not correct.

• The central legislature was supposed to have been given very wide powers. It could legislate
for the whole of British India, for the Indian subjects and the servants of the government, whether
inside or outside the country. It could repeal or amend any law already existing in the country. The
members were given the right to move resolutions and motions for adjournment of the House, to
consider urgent questions of public importance immediately. They had the right to ask questions and
supplement. Short notice questions could also be asked. The members enjoyed the right of freedom
of speech. So, statement 3 is correct.

Therefore, option (c) is the correct answer.

Subject: History | The British Administrative reforms

Tags: Factual

Q74. Consider the following statement with reference to Dr BR Ambedkar:

1. He led the Nasik Satyagraha in 1930 to establish the right of the untouchables for temple entry.

2. He launched a peasant agitation against the Khoti system.

3. He initiated a campaign in support of Mahar watan with the help of Indian National Congress.

4. He founded the Independent Labour Party (ILP) to advance the welfare of the labouring classes.

Which of the statements given above is/are correct?

a) 1 and 3 only

b) 2 and 3 only

c) 1, 2 and 4 only

d) 1, 2, 3 and 4

Answer: c

Explanation:

• Dr B.R Ambedkar led a satyagraha in March 1930 to establish the right of the Untouchables
to enter the famous temple of Kalaram at Nasik. In 1930, at Nasik, the Depressed Classes had formed
a Satyagraha Committee. Sporadic acts of violence occurred between the demonstrators and the
upper castes. Then, the latter, contrary to a newly reached agreement, prevented some Mahars
from pulling the processional chariot during the annual temple festival. This incident reinforced
Ambedkar’s determination

but he eventually dissociated himself from the movement in 1934. So, statement 1 is correct.

• In the 1920s he initiated his campaign against the Mahar watan, a practice which was
continued under British rule.

• He explained Mahar watan problems as the following:

o In Mahar watan, Mahars don’t have fixed work but have to do various kinds of works. Mahar
watan is like veth begar where they have to do all the work.

o In Mahar watan the number of people who have to work in one village is not decided. The
time on work is not decided; a person has to work at any given time whenever the village
administrative and upper caste group asks him to do so. In the case of Mahars the officiator is the
person whose name is entered in what is called the service register and he is not the only one
person who is liable to render service to the government, but his whole family is liable to render
service to the government. As a matter of fact, every department claims their services. So,
statement 3 is not correct.

• Within the Bombay presidency, the khoti system existed separately in the Konkan region.
The khots (landlords) rented villages and farm land and collected villages’ revenues for the
government as hereditary officers. The colonial government contributed to the consolidation of the
khoti land tenure system and perpetuated its exploitative forms. The khots were mostly Chitpavan
Brahmins and a few were high-caste Marathas and Muslims. Khots could not only keep a part of the
revenue accruement, but often set themselves up as local minor Rajahs.

• During the 1930s, Ambedkar launched a peasant agitation against the khoti. One of the main
aims of the formation of the Independent Labour Party (ILP) in 1936 was to concentrate on and solve
the problems and grievances of landless and poor tenants, agriculturalists and workers. So,
statement 2 and statement 4 are correct.

Therefore, option (c) is the correct answer.

Subject: History | Important personalities

Tags: Factual

Q75. Which among the following factors can have a positive impact on India’s Balance of Trade?

1. Depreciation of rupee value against US Dollar

2. Imposing Minimum Import Price on raw materials

3. Adopting contractionary monetary policy by the RBI

4. Increase in external borrowing by the Government

Select the correct answer using the code given below.

a) 1 and 2 only

b) 3 and 4 only
c) 1, 2 and 3 only

d) 1, 2, 3 and 4

Answer: a

Explanation:

• The balance of trade is the value of a country's exports minus its imports. A positive trade
balance (surplus) is when exports exceed imports; while a negative trade balance (deficit) is when
exports are less than imports.

• Depreciation in the domestic currency is expected to give a boost to exports, as traders get
more local currency when they convert the export proceeds. Hence, it would lead to a positive
Balance of Trade. So, statement 1 is correct.

• Minimum import price (MIP) is a temporary measure to provide protection to domestic


farmers from predatory pricing of imports. Below this price, import of the commodity is not allowed.
Imposing a minimum import price on goods could help in curbing cheaper imports, which would lead
to positive Balance of Trade Balance.So, statement 2 is correct.

• Contractionary monetary policy is enacted to halt exceptionally high inflation rates or


normalize the effects of expansionary policy. Tightening the money supply discourages business
expansion and consumer spending and negatively impacts exporters, which can reduce aggregate
demand. So, statement 3 is not correct.

• Increase in gross borrowings from external markets would free up additional liquidity in the
domestic market. It will lead to appreciation of the value of Indian rupee, consequently hurting the
country's exports. So, statement 4 is not correct.

Therefore, option (a) is the correct answer.

Relevance: According to reports, the Balance of Trade in India averaged -3.02 USD Billion from 1957
until 2022, reaching an all-time high of 0.79 USD Billion in June of 2020 and a record low of -22.91
USD Billion in November of 2021.

Subject: Economics | Balance of payment and Foreign reserves

Tags: Conceptual, Analytical

Q76. Which of the following factors can lead to an increase in Demand-pull inflation?

1. Increase in crude oil prices due to prevailing tensions in Ukraine.

2. State Bank of India raising its interest rates on personal loans.

3. Appreciation in the value of Indian Rupee against US Dollars.

4. Providing Universal Basic Income for all Indian citizens.

Select the correct answer using the code given below.

a) 1 and 3 only

b) 2, 3 and 4 only
c) 4 only

d) 1, 2, 3 and 4

Answer: c

Explanation:

• Demand-pull inflation is a period of inflation which arises from rapid growth in aggregate
demand. If aggregate demand rises faster than productive capacity, then firms will respond by
putting up prices, creating inflation. Cost-push inflation occurs when prices rise due to higher costs
of production and higher costs of raw materials. Cost-push inflation is determined by supply-side
factors, such as higher wages and higher oil prices.

• Increase in crude oil prices due to shortage of supply due to prevailing tensions in petroleum
exporting countries, leads to cost-push inflation. So, point 1 is not correct.

• Increase in interest rates of loans by a bank leads to decrease in the volume of disposable
income with the people. This in turn leads to decreased money supply thereby reducing demand.
This leads to reduction in levels of demand pull inflation. So, point 2 is not correct.

• An appreciation in the exchange rate will tend to reduce cost-push inflation, as prices of
imported products gets cheaper. So, point 3 is not correct.

• Providing Universal Basic Income for all citizens leads to an increase in money supply in the
economy. This in turn leads to demand pull inflation. So, point 4 is correct.

Therefore, option (c) is the correct answer.

Relevance: According to Economists, the prevailing tensions in Ukraine due to Russian invasion is
projected to cause ‘Demand-Pull Inflation’ in India.

Subject: Economy | Inflation

Tags: Analytical, Conceptual

Q77. Consider the following statements with reference to Gross Domestic Product Deflator:

1. It measures the average change in selling prices received by domestic producers of intermediate
goods and services over time.

2. Changes in consumption patterns are automatically reflected in the deflator.

Which of the statements given above is/are correct?

a) 1 only

b) 2 only

c) Both 1 and 2

d) Neither 1 nor 2

Answer: b

Explanation:
• The GDP deflator, also called implicit price deflator, is a measure of inflation. It is the ratio of
the value of goods and services an economy produces in a particular year at current prices to that of
prices that prevailed during the base year. The producer price index is a family of indexes that
measures the average change in selling prices received by domestic producers of intermediate goods
and services over time. The PPI measures price changes from the perspective of the seller and differs
from the CPI which measures price changes from the perspective of the buyer. So, statement 1 is not
correct.

• The fixed basket used in Consumer Price Index (CPI) calculations is static and sometimes
misses changes in prices of goods outside of the basket of goods. Since GDP isn't based on a fixed
basket of goods and services, the GDP price deflator has an advantage over the CPI.

• Changes in consumption patterns or the introduction of new goods and services are
automatically reflected in the deflator but not in the CPI. So, statement 2 is correct.

• The GDP deflator, being a weighted average of Wholesale Price Index (WPI) and CPI, has
given higher weightage to the WPI, than the CPI.

Therefore, option (b) is the correct answer.

Relevance: The Union Budget 2022 targeted a nominal GDP growth of 11.1 per cent in 2022-23 with
a GDP deflator of 3.0-3.5 per cent.

Subject: Economy | Inflation

Tags: Conceptual, Analytical

Q78. Consider the following statements with reference to Direct Benefit Transfer (DBT) in India:

1. The use of the Public Financial Management System portal is mandatory for payments under DBT.

2. Only the beneficiaries, whose bank accounts are linked with their Aadhaar, are eligible to get
benefits under DBT.

3. DBT Mission and all matters related to it have been placed under the Union Ministry of Finance.

Which of the statements given above is/are correct?

a) 1 only

b) 2 and 3 only

c) 1 and 3 only

d) 1, 2 and 3

Answer: a

Explanation:

• Transferring subsidies directly to the people through their bank/Post office account is Direct
Benefit Transfer. It aims to timely transfer of benefit to the citizen by bringing efficiency,
effectiveness, transparency and accountability in the Government system. Through the DBT, the
Government intends to achieve electronic transfer of benefits, reduce delays in payments and most
importantly, accurate targeting of beneficiaries, thereby curbing leakages and duplication. With the
aim of reforming Government delivery system by re-engineering the existing process in welfare
schemes for simpler and faster flow of information/funds and to ensure accurate targeting of the
beneficiaries, de-duplication and reduction of fraud Direct Benefit Transfer (DBT) was started on 1st
January, 2013.

• The use of the Public Financial Management System (PFMS) has been made mandatory for
payment, accounting and reporting under Direct Benefit Transfer, with effect from 1st April 2015.
So, statement 1 is correct.

o The Public Financial Management System (PFMS) is a web-based online software application
developed and implemented by the Controller General of Accounts (CGA), Department of
Expenditure, Ministry of Finance, Government of India. PFMS started during 2009 with the objective
of tracking funds released under all Plan schemes of Government of India, and real time reporting of
expenditure at all levels of Programme implementation.

o The primary function of PFMS today is to facilitate sound Public Financial Management
System for Government of India by establishing an efficient fund flow system as well as a payment
cum accounting network. PFMS provides various stakeholders with a real time, reliable and
meaningful management information system and an effective decision support system, as part of
the Digital India initiative of Government of India.

o No payments under the Direct Benefit Transfer schemes (except PAHAL) are to be
processed, unless the electronic payment files for such payments are received through the PFMS
system developed by the CGA from 1st April, 2015.

• According to the Reserve Bank of India guidelines, use of Aadhaar cards and seeding of bank
accounts with those numbers are purely voluntary and not mandatory. Aadhaar is not mandatory in
DBT schemes. Since Aadhaar provides unique identity and is useful in targeting the intended
beneficiaries, Aadhaar is preferred and beneficiaries are encouraged to have Aadhaar. So, statement
2 is not correct.

• DBT Mission was created in the Planning Commission to act as the nodal point for the
implementation of the DBT programmes. The Mission was transferred to the Department of
Expenditure in July, 2013 and continue to function till 14.9.2015. To give more impetus, DBT Mission
and matters related thereto has been placed in Cabinet Secretariat under Secretary (Co-ordination &
PG) w.e.f. 14.9.2015. So, statement 3 is not correct.

Therefore, option (a) is the correct answer.

Relevance: The Union Power Minister recently told the Rajya Sabha that there was no proposal to
make Direct Benefit Transfer (DBT) mandatory for power consumers.

Subject: Economy | Money and Banking

Tags: Conceptual, Analytical

Q79. Consider the following statements with reference to Forward Exchange Rate:

1. It is the rate of exchange for a future dated foreign exchange market transaction.

2. Its adoption helps in avoiding the exchange rate risks of foreign currencies.
3. Forward premium occurs when the forward exchange rate is quoted lower than the spot exchange
rate.

Which of the statements given above are correct?

a) 1 and 2 only

b) 2 and 3 only

c) 1 and 3 only

d) 1, 2 and 3

Answer: a

Explanation:

• The forward exchange rate is the rate of exchange, agreed upon now, for a foreign exchange
market transaction that will occur at a specified date in the future. The agreement to make such an
exchange in the future at a rate agreed upon now is called a forward contract. So, statement 1 is
correct.

• Forward Rates = spot rate +/- premium/discount.

• The essential idea of entering into a forward contract is to fix the exchange rate in advance
and thereby avoid the exchange rate risk. So, statement 2 is correct.

• Forward premium occurs when the forward exchange rate is quoted higher than the spot
exchange rate. The expectation of the market is that the domestic currency will be worth less in the
future or will depreciate in value versus the foreign currency. So, statement 3 is not correct.

Therefore, option (a) is the correct answer.

Relevance: Recently, the one-month USD-INR forward rate increased higher than one- and two-year
forward ra¬tes, an anomaly bro¬ught on by arbitrageurs playing between the offshore and onshore
markets.

Subject: Economics | Balance of payment and Foreign reserves

Tags: Conceptual, Analytical

Q80. Consider the following statements with reference to Performance Budgeting:

1. Funding is based on program efficiency and necessity rather than budget history.

2. It was first introduced in 2015 in the Central Ministries in India.

Which of the statements given above is/are correct?

a) 1 only

b) 2 only

c) Both 1 and 2

d) Neither 1 nor 2
Answer: d

Explanation:

• Performance budgeting is a system of presenting public expenditure in terms of functions


and programmes reflecting the government output and its cost; whereas, Zero-based budgeting
(ZBB) is a budgeting process that allocates funding based on program efficiency and necessity rather
than budget history. So, statement 1 is not correct.

• The government is obligated to report on "what is done" and "how much is done" for the
good of the people in the Performance Budget. The 'Outcome Budget' is how the Performance
Budget is referred to in India.It is designed to serve the purposes of long range planning.

• A 'Performance Budget' is one in which the outcome of an activity is used as the basis for a
budget. The performance budget was created in the United States for the first time ever in 1949.The
Central government announced its decision to introduce performance budgeting in four central
ministries in 1968. In 1977-78, about 32 developmental departments in the Central government
introduced performance budgeting in their selected units. So, statement 2 is not correct.

Therefore, option (d) is the correct answer.

Relevance: The Union Budget 2022-23, along with the Performance Budget was recently tabled in
the Parliament.

Subject: Economics | public finance

Tags: Conceptual, Factual

Q81. Consider the following statements with reference to Global Minimum Tax:

1. It was initiated by International Monetary Fund to eliminate the practice of shifting profits to tax
havens.

2. The Global Anti-Base Erosion provides a framework for a minimum of 15% taxation of
multinational companies.

Which of the statements given above is/are correct?

a) 1 only

b) 2 only

c) Both 1 and 2

d) Neither 1 nor 2

Answer: b

Explanation:

• The global minimum tax was agreed in October 2021 between countries who are members
of the Organisation for Economic Co-operation and Development (OECD)/G20 inclusive framework
on tax base erosion and profit shifting (BEPS). So, statement 1 is not correct.
• It is designed to ensure that multinational enterprises (MNEs) will be subject to a minimum
15% tax rate in every country of operation from 2023, thus preventing situations where these MNEs
take advantage of differing tax regimes between jurisdictions and effectively avoid paying tax.

• The Pillar Two model rules define the scope and set out the mechanism for the so-called
Global Anti-Base Erosion (GloBE) rules under Pillar Two, which will introduce a global minimum
corporate tax rate set at 15%. So, statement 2 is correct.

Therefore, option (b) is the correct answer.

Relevance: A group of 136 countries, including India, in October 2021 agreed to set a minimum
global tax rate of 15% for big companies.

Subject: Economics | International Organizations(eco)

Tags: Conceptual

Q82. Consider the following statements with reference to the Line of Credit (LOC) extended by the
Government of India:

1. It is a one-time development grant provided to developing countries by the Indian Government.

2. Under it, goods and services for at least 75% of the value of the contract must be sourced from
India.

3. The full amount extended through LOCs should exclusively be on a Government to Government
basis.

4. Bangladesh has been the recipient of the largest concessional credit given by India to any single
country.

Which of the statements given above are correct?

a) 1, 2 and 3 only

b) 2 and 4 only

c) 3 and 4 only

d) 1, 2, 3 and 4

Answer: b

Explanation:

• A Line of Credit is a financing mechanism through which Exim Bank extends support for
export of projects, equipment, goods and services from India. The Line of Credit is not a grant but a
‘soft loan’ provided on concessional interest rates to developing countries, which has to be repaid by
the borrowing government. So, statement 1 is not correct.

• The LOCs also helps to promote exports of Indian goods and services, as 75% of the value of
the contract must be sourced from India. So, statement 2 is correct.

• Till 2003-04, the LOCs were from Government to Government. Accordingly, the full amount
covered by the LOCs, used to be provided in the Budget. Since 2003-04, this system has been
substituted by extending GOI supported Lines of Credit through Exim Bank of India. So, statement 3
is not correct.

• Out of the total LOCs of US$ 30.59 billion, US$ 16.095 billion have been extended to Asian
countries, with the largest value of commitments having been made in India’s immediate
neighbourhood. The largest concessional credit given by India to any single country has been to
Bangladesh. So, statement 4 is correct.

Therefore, option (b) is the correct answer.

Relevance: The RBI recently issued a notification to operationalize the USD 500-million line of credit
provided by India to Sri Lanka, which is facing the worst economic crisis in decades due to the
pandemic, to buy petroleum products.

Subject: Economics | Balance of payment and Foreign reserves

Tags: Conceptual, Factual

Q83. Which one of the following best explains the term ‘Economies of scale’?

a) Decrease in costs of production due to increase in number of different goods produced.

b) Decrease in costs of production due to increase in production volume.

c) Decrease in costs of production due to limited market size and large number of competitors.

d) Decrease in costs of production due to extensive fiscal support and financial benefits.

Answer: b

Explanation:

• Economies of scale are reductions in average costs attributable to production volume


increases. They typically are defined in relation to firms, which may seek to achieve economies of
scale by becoming large or even dominant producers of a particular type of product or service.

• They are different from Economies of scope, which means the reductions in average costs
attributable to an increase in the number of goods produced.

Therefore, option (b) is the correct answer.

Relevance: According to officials, farmer producer organisations in India, which have farmers as the
share- holders, would leverage economies of scale which would lead to reduction in cost of
production and enhance farmers’ incomes.

Subject: Economics | Industry

Tags: Conceptual

Q84. Consider the following statements with reference to ‘Velocity of circulation’:

1. It is the magnitude by which base money is amplified in the banking system to the stock of broad
money.
2. It has the same effect on the economy, similar to that of the Money multiplier effect.

3. It is directly proportional to inflation levels in an economy.

Which of the statements given above is/are correct?

a) 1 and 2 only

b) 2 and 3 only

c) 3 only

d) 1, 2 and 3

Answer: c

Explanation:

• Velocity of circulation is the amount of units of money circulated in the economy during a
given period of time. So, statement 1 is not correct.

• It is the number of times the available money stock may roll over or change hands to finance
transactions equivalent to nominal GDP. It is measured by dividing GDP by the country's total money
supply.

• Velocity is different from the money multiplier. The money multiplier is the link between
“broad money” and “reserve money”, whereas velocity is the link between “broad money” and
“nominal GDP”. So, statement 2 is not correct.

• A high velocity of circulation in a country indicates a high degree of inflation, which means
that the Velocity of circulation is directly proportional to inflation levels in an economy. So,
statement 3 is correct.

Therefore, option (c) is the correct answer.

Relevance: According to Economists, due to COVID-19 lockdowns in India, the velocity of circulation
of money had impacted the levels of inflation in India.

Subject: Economy | Money and Banking

Tags: Conceptual, Analytical

Q85. Which of the following receipts are classified under the category of non-debt capital receipts of
the Union Government?

1. Proceedings from strategic disinvestment

2. Issuance of Special Securities to Public Sector Banks

3. Issue of bonus shares to investors of government companies

4. Grants-in-aid from international financing institutions

Select the correct answer using the code given below.

a) 1 and 3 only
b) 2 and 4 only

c) 1, 3 and 4 only

d) 1, 2, 3 and 4

Answer: a

Explanation:

• Capital receipts are receipts that create liabilities or reduce financial assets. They also refer
to incoming cash flows. Capital receipts can be both non-debt and debt receipts.

• The Non debt capital receipts (NDCR) of the union government include:

o Recoveries of loans and advances given to state governments, Union territories and foreign
governments

o Disinvestment proceeds

o Money accrued to the Union government from listing of central government companies and
issue of bonus shares.

o Strategic disinvestment

o Listing of PSUs in stock markets and

o Issue of bonus shares. So, point 1 and point 3 are correct.

• Issuance of Special Securities to Public Sector Banks is a part of debt capital receipts of the
union government. So, point 2 is not correct.

• Grants-in-aid received from international financing institutions form a part of Non-Tax


Revenue of the government. So, point 4 is not correct.

Therefore, option (a) is the correct answer.

Relevance: The Union Budget 2022-23 has projected that the non-debt capital receipts of the Union
Government would be around Rs 79291 crores.

Subject: Economics | Public Finance

Tags: Conceptual, Analytical

Q86. Consider the following statements with reference to Global Depository Receipts (GDR):

1. Indian companies can get listed on foreign exchanges through a GDR.

2. They are considered to have less liquidity than the American Depository Receipts.

3. They form a part of Foreign Direct Investment in India.

4. As of now, there is no approved Global Depository functioning in India.

Which of the statements given above is/are correct?

a) 1, 2 and 3 only

b) 2 and 4 only
c) 3 only

d) 1, 2, 3 and 4

Answer: a

Explanation:

• Depository Receipts (DRs) are negotiable securities issued outside India by a Depository
bank, on behalf of an Indian company, which represent the local Rupee denominated equity shares
of the company held as deposit by a Custodian bank in India. DRs are traded on Stock Exchanges in
the US, Singapore, Luxembourg, London, etc. DRs listed and traded in the US markets are known as
American Depository Receipts(ADRs) and those listed and traded elsewhere are known as Global
Depository Receipts (GDRs).GDR is the only way through which Indian firms can make their shares
available on different foreign exchanges.So, statement 1 is correct.

• GDRs are considered to have less liquidity than the American Depository Receipts. So,
statement 2 is correct.

• The inward remittance received by the Indian company vide issuance of Depository Receipts
are treated as Foreign Direct Investment (FDI) and counted towards FDI. So, statement 3 is correct.

• The International Financial Services Centre in Gujarat allows Indian firms to list their global
receipts. Companies can now raise funds through foreign sources. So, statement 4 is not correct.

Therefore, option (a) is the correct answer.

Relevance: Recently, the SEBI passed directions against Southern Ispat and Energy, its officials and
certain other individuals and entities in a matter pertaining to manipulation in issuance of global
depository receipts (GDR).

Subject: Economics | Investment

Tags: Conceptual

Q87. Consider the following statements with reference to Reserve Tranche Position (RTP) of the
International Monetary Fund (IMF):

1. It grants unconditional drawing rights to the member countries.

2. Member countries can draw interest free advances out of the RTP during Balance of Payment
crises.

3. The share of RTP in India’s foreign exchange reserves is higher than that of the Special Drawing
Rights (SDR).

Which of the statements given above is/are correct?

a) 1 and 2 only

b) 2 and 3 only

c) 3 only

d) 1, 2 and 3

Answer: a
Explanation:

• The reserve tranche is a segment of an International Monetary Fund (IMF) member


country’s quota that is accessible without fees or economic reform conditions.

• The reserve tranches that countries hold with the IMF are considered their facilities of first
resort, meaning they will tap into them before seeking a formal credit tranche that charges interest.

• It is an international reserve asset that represents a member’s automatic (unconditional)


drawing right on the Fund upon representation of a balance of payments need, created by the
payment of the foreign exchange component of the quota subscription and capable of being
expanded by the Fund’s use of the member’s currency in its transactions with other member
countries. So, statement 1 is correct.

• Part of a country’s quota can be withdrawn from the IMF without any interest during critical
situations of a country such as Balance of Payment (BOP) crises. So, statement 2 is correct.

• The largest value of India’s foreign exchange reserves is held in Foreign Currency Assets,
followed by Gold, Special Drawing Rights (SDRs) and Reserve Tranche Position (RTP).So, statement 3
is not correct.

Therefore, option (a) is the correct answer.

Relevance: The RBI had recently released the statistics on India’s foreign exchange reserves and its
compositions.

Subject: Economics | International Organizations(eco)

Tags: Conceptual, Analytical

Q88. Consider the following statements:

1. Hallmarking is mandatory for selling diamond jewellery in India.

2. Gold watches and fountain pens are exempted from mandatory hallmarking.

3. The standards for hallmarking for jewellery are prescribed by the India Bullion and Jewellers
Association.

Which of the statements given above is/are correct?

a) 1 only

b) 2 only

c) 1 and 3 only

d) 1, 2 and 3

Answer: b

Explanation:

• Hallmarking is the accurate determination and official recording of the proportionate


content of precious metal in precious metal articles. Hallmarks are thus official marks used in many
countries as a guarantee of purity or fineness of precious metal articles.
• With effect from 16 June 2021, Hallmarking was made mandatory for gold jewellery/
artefacts in only in 256 districts in India and for jewellers having annual turnover above Rs 40 lakh.
The government notified two categories—gold jewellery and gold artefacts; and silver jewellery and
silver artefacts—under the purview of hallmarking. So, hallmarking in India is available for jewellery
of only two metals—gold and silver. So, statement 1 is not correct.

• The gold watches, fountain pens and special types of jewellery such as Kundan, Polki and
Jadau are exempted from hallmarking in India. So, statement 2 is correct.

• The Bureau of Indian Standards (BIS), a statutory body functioning under Department of
Consumer affairs, Ministry of Consumer Affairs, Food & Public Distribution, Government of India
fixes the standards for hallmarking jewellery in India. So, statement 3 is not correct.

Therefore, option (b) is the correct answer.

Relevance: With effect from 16 June 2021, Hallmarking was made mandatory for gold jewellery/
artefacts in only in 256 districts in India and for jewellers having annual turnover above Rs 40 lakh.

Subject: Economics | Industry

Tags: Conceptual

Q89. Which of the following best explains the term ‘deferred tax’?

a) It is a form of tax evasion, which leads to significant revenue loss to the Government.

b) It is the postponement of paying tax by the companies or individuals to a future date.

c) It is the income or transaction which is free from taxation at a later date at the Union, state, or
local level.

d) It is the tax paid to the government by the payer of the income rather than by the recipient.

Answer: b

Explanation:

• Deferred tax is the tax which is estimated for the current period or is due for that term, but
it has not been paid yet. The delay or the postponement comes when there is a difference in time
from when the tax is accumulated and when the tax is actually paid.

• It is calculated as the company's anticipated tax rate times the difference between its
taxable income and accounting earnings before taxes. Deferred tax liability is the amount of taxes a
company has "underpaid" which will be made up in the future. This doesn't mean that the company
hasn't fulfilled its tax obligations. Rather it recognizes a payment that is not yet due.

• For example, a company that earned net income for the year knows it will have to pay
corporate income taxes. Because the tax liability applies to the current year, it must reflect an
expense for the same period. But the tax will not actually be paid until the next calendar year. In
order to rectify the accrual/cash timing difference, tax is recorded as a deferred tax liability.

Therefore, option (b) is the correct answer.


Relevance: The government-owned Oil and Natural Gas Company (ONGC) has opted for the lower
tax of 22% resulting in a decrease of Rs 1,304 crore in current taxes, and a reduction of Rs 8,541
crore in deferred taxes.

Subject: Economics | Public Finance

Tags: Conceptual

Q90. Which of the following factors contribute to the pricing of ‘Liquefied Petroleum Gas’ (LPG) in
India?

1. Import Parity Price

2. Currency Exchange Rate

3. Goods and Service Tax

4. Inland Freight

Select the correct answer using the code given below.

a) 2 and 3 only

b) 1 and 4 only

c) 1, 3 and 4 only

d) 1, 2, 3 and 4

Answer: d

Explanation:

• LPG pricing in India is done on the basis of a formula — import parity price (IPP). The IPP is
determined based on LPG prices in the international market, assuming that the fuel is imported into
the country.

o The IPP, based on Saudi Aramco’s LPG price, includes the FOB (free on board) price, ocean
freight, insurance, custom duties, port dues, etc. This price, quoted in dollars, is then converted to
rupees. To this is added the cost of inland freight, marketing costs and margins charged by the oil
companies, bottling charges, dealer commission and the GST. This gives the retail selling price of the
non-subsidised LPG cylinder for the Indian customer. The price of LPG cylinders in India is reset on a
monthly basis, effective from the first of every month

• The Import Parity Price (IPP) is the price at the border of a good that is imported, which
includes international transport costs and tariffs. If a good is cheaper abroad, i.e. the domestic price
is higher than the IPP; traders have a strong incentive to import the good.

Therefore, option (d) is the correct answer.

Relevance: The recent hike in LPG prices will impact household inflation expectations, and could
further hit subdued consumption sentiment.

Subject: Economy | Inflation

Tags: Factual
Q91. With reference to ‘oil seeds crops’ in India, consider the following statements:

1. More than fifty percent of the total oilseeds area is confined to irrigation farming.

2. Soybean accounts for maximum oil seed production.

3. Castor seed can be cultivated as both as rabi and kharif crop.

Which of the statements given above are correct?

a) 1 and 2 only

b) 2 and 3 only

c) 1 and 3 only

d) 1, 2 and 3

Answer: b

Explanation:

• There are two major sources of Oilseeds i.e. Primary and Secondary. Primary sources are
made combining the edible group [(Groundnut, Rapeseed (Toria, Mustard and Sarson), Soybean,
Sunflower, Sesame, Safflower and Niger)] and non-edible group (Castor and Linseed). Similarly,
secondary sources have been established combining edible group (Seasonal crops : Cottonseed, Rice
bran, Maize germ, Watermelon, Plantation crops : Coconut, Red-oil palm, Tree borne oilseeds: Sal
seed, Mahua, Mango-kernel, Cheura / Phulwara, Kokum, Dhupa, Simarouba) and non-edible group
(Seasonal crops : Mesta seed, Tobacco seed, Plantation crops : Rubber – seed, other Tree Borne
Oilseeds: Neem, Karanj, Pilu or Khakan, Palash, Nahor, Undi, Pisa, Wild-apricot, Rattan-jyot, Maroti,
Jojoba, etc.).

• Almost 72% of the total oilseeds area is confined to rainfed farming cultivated mostly by
marginal and small farmers. So, statement 1 is not correct.

• India has been cultivating almost all oilseed crops. Major ones are Soybean, Groundnut,
Rapeseed-Mustard, Sesame, Sunflower, Castor, Safflower, Linseed and Niger. Among the nine
oilseed crops in India, the highest average contribution to total production of oilseeds is of soybean
(38%) followed by Rapeseed-Mustard (27%) and groundnut (27%) (Average of 2016-17 to 2020-21.
Similarly, highest average area contribution to total oilseed area is of soybean (44%) followed by
Rapeseed-Mustard (24%) and Groundnut (20%). On an average, total Kharif oilseed crops
contribution to total production is about 67% and the remaining 33% is of Rabi / summer oilseed
crops. So, statement 2 is correct.

• Sesamum is a Kharif crop in the north and a Rabi crop in south India. Castor seed is grown
both as Rabi and Kharif crop. So, statement 3 is correct.

Therefore, option (b) is the correct answer.

Subject: Economy| Agriculture and allied sector

Tags: Factual
Q92. Consider the following statements with reference to Current Account Deficit (CAD):

1. Depreciation in the value of Indian rupee against US Dollar has a positive impact on India’s CAD.

2. Liberalising External Commercial Borrowing norms for corporates can directly lead to reduction in
India’s CAD.

3. Insurances paid on Indian exports are included under invisibles while calculating CAD.

4. The Fiscal Responsibility and Budget Management Act, 2003 provides a statutory framework for
reducing CAD in India.

Which of the statements given above are correct?

a) 1 and 3 only

b) 2 and 4 only

c) 1, 3 and 4 only

d) 1, 2, 3 and 4

Answer: a

Explanation:

• The Current Account Deficit (CAD) is a measurement of a country’s trade where the value of
the goods and services it imports exceeds the value of the products it exports.

• Depreciation in the value of Indian Rupee against US Dollar leads to an increase in exports
from India. This in turn reduces the trade deficit, thereby having a positive impact on CAD. So,
statement 1 is correct.

• External commercial borrowing is included under the category of Capital Account. So it does
not directly lead to a reduction in India’s CAD. So, statement 2 is not correct.

• Invisibles are broadly classified under three heads, viz., services, transfers and income. While
services comprise travel, transportation, insurance, the government not included elsewhere (GNIE)
and miscellaneous (i.e. other services); transfers constitute private transfers and official transfers
and income includes investment income and compensation of employees.‘Insurance receipts’
consist of insurance on exports, premium on life and non-life policies and reinsurance premium from
foreign insurance companies. Insurance on exports is directly related to total exports from India.So,
statement 3 is correct.

• The Fiscal Responsibility and Budget Management Act (FRBM Act), 2003, establish financial
discipline to ensure optimum revenue deficit, fiscal deficit, tax revenue and the total outstanding
liabilities only. It does not mandate reduction commitments for CAD.So, statement 4 is not correct.

Therefore, option (a) is the correct answer.

Relevance: According to the data of Reserve Bank of India, India’s Current Account Deficit (CAD)
increased to $23 billion (2.7 per cent of GDP) in the third quarter (Q3) of 2021-22 from $9.9 billion
(1.3 per cent of GDP) in Q2 of 2021-22 and $2.2 billion (0.3 per cent of GDP) in Q3 of 2020-21.

Subject: Economics | Public Finance

Tags: Conceptual, Analytical


Q93. Consider the following statements with reference to Gross Domestic Product (GDP) calculation
in India:

1. The effects of trade surplus and deficits are not reflected in the GDP calculations in India.

2. Narrowing differences between GDP and Net Domestic Product (NDP) represents technology
advancements in an economy.

3. The Private Final Consumption Expenditure constitutes the major share in the Expenditure
Component of GDP.

4. The current base year for GDP calculation in India is 2020-2021.

Which of the statements given above are correct?

a) 1, 2 and 3 only

b) 2 and 3 only

c) 1 and 4 only

d) 1, 2, 3 and 4

Answer: b

Explanation:

• Gross Domestic Product (GDP) is the value of all final goods and services produced within
the boundary of a nation during one year period.

• It is also calculated by adding national private consumption, gross investment, government


spending and trade balance (exports-minus-imports). The use of the exports-minus-imports factor
removes expenditures on imports not produced in the nation, and adds expenditures of goods and
services produced which are exported, but not sold within the country. Hence, the effects of trade
surplus/deficits are also reflected in the GDP calculations in India.So, statement 1 is not correct.

• Net Domestic Product (NDP) is the GDP calculated after adjusting the weight of the value of
‘depreciation’. This is, basically, net form of the GDP, i.e., GDP minus the total value of the ‘wear and
tear’ (depreciation) that happened in the assets while the goods and services were being produced.
NDP = GDP – Depreciation. NDP of an economy has to be always lower than its GDP for the same
year, since there is no way to cut the depreciation to zero. But due to the development of several
techniques and tools such as ‘ball-bearing’, ‘lubricants’, etc., the loss due to depreciation are being
reduced to some extent. Hence a narrow gap between GDP and NDP reflects advancements in
technologies in an economy. So, statement 2 is correct.

• India’s GDP is calculated with two different methods, one based on economic activity (at
factor cost), and the second on expenditure (at market prices).

• In the Estimates of Expenditure Components of GDP, the Private Final Consumption


Expenditure constitutes the largest share. So, statement 3 is correct.

• The current base year for GDP calculations on constant prices is 2011-12. The Government
had recently clarified that it has no plans to change the base year to 2020-2021 for GDP calculations.
So, statement 4 is not correct.
Therefore, option (b) is the correct answer.

Relevance: According to the recent written reply of minister of state (Independent charge) of the
Ministry of Statistics and Programme Implementation in the Lok Sabha, the government had no
plans to change the base year to 2020-2021 for Gross Domestic Product (GDP) calculations.

Subject: Economics | public finance

Tags: Conceptual, Analytical

Q94. Which of the following functions are performed by Micro ATMS?

1. Cash Deposits

2. Cash Withdrawal

3. Fund Transfer

4. Balance Enquiry

Select the correct answer using the code given below.

a) 1 and 4 only

b) 2 and 3 only

c) 1, 3 and 4 only

d) 1, 2, 3 and 4

Answer: d

Explanation:

• Micro ATM is a device that is used by a million Business Correspondents to deliver basic
banking services. The platform will enable Business Correspondents (who could be a local kirana
shop owner and will act as ‘micro ATM’) to conduct instant transactions.

• The micro platform will enable function through low cost devices (micro ATMs) that will be
connected to banks across the country. This would enable a person to instantly deposit or withdraw
funds regardless of the bank associated with a particular BC. This device will be based on a mobile
phone connection and would be made available at every BC. Customers would just have to get their
identity authenticated and withdraw or put money into their bank accounts. This money will come
from the cash drawer of the BC. Essentially, BCs will act as bank for the customers and all they need
to do is verify the authenticity of customer using customers’ UID. The basic transaction types, to be
supported by micro ATM, are Deposit, Withdrawal, Fund transfer and Balance enquiry.

Therefore, option (d) is the correct answer.

Relevance: Micro ATMs usage tripled during pandemic, outpacing regular ATMs

Subject: Economy | Money and Banking

Tags: Factual

Q95. Which of the following can be caused by an Earthquake?

1. Volcanic eruption
2. Lahars

3. Landslide

4. Liquefaction

Select the correct answer using the code given below.

a) 1, 2 and 4 only

b) 1 and 3 only

c) 2, 3 and 4 only

d) 1, 2, 3 and 4

Answer: d

Explanation:

• Earthquakes are one of the most devastating and frightening natural disasters a person can
experience. They happen without warning in areas all around the world. Earthquakes can cause
major damage and fatalities in populated areas, but the earthquake itself is not always to blame.
Other natural disasters can be caused by earthquakes and these can be equally, and sometimes
more, destructive.

• Earthquakes may trigger volcanic eruptions. Most earthquakes occur on or near the edges of
tectonic plates. Similarly, a volcano is the result of the interaction of these plates. Scientists believe
that seismic waves coming from earthquakes cause disturbances in the molten rock beneath
volcanoes, making them active. So, point 1 is correct.

• The deadliest types of mudslides are called lahars. These are almost always associated with a
volcano and can be incredibly destructive. Lahars occur in most cases when volcanic activity causes a
dramatic and sudden melting of snow and ice around a volcano. The lahar will gain the consistency
of concrete and be very powerful, annihilating everything in its path and then finally covering
whatever area it ends up at too great depths. So, point 2 is correct.

• When the Earth moves during an earthquake, a landslide or avalanche can occur. Any area
that has the right conditions, including moisture and the angle of the slope, can potentially
experience these natural disasters. When the Earth shakes, debris, soil or snow on a hilltop or
mountainside has the potential of sliding. So, point 3 is correct.

• Liquefaction can happen following an earthquake. It is the mixing of sand or soil and
groundwater (water underground) during the shaking of a moderate or strong earthquake. The
ground turns into a quicksand consistency when water is mixed with it. If a building is built upon this
type of ground, it can tip, fall over and even sink. So, point 4 is correct.

Therefore, option (d) is the correct answer.

Relevance: Recently many earthquakes were reported from different parts of India.

Subject: Geography| Natural Hazards and Disasters

Tags: Conceptual
Q96. Fujiwhara effect, often seen in the news, is related to which one of the following?

a) Funnelling effect created by the mountains for monsoon winds

b) A binary interaction between two nearby cyclonic vortices

c) Blocking of sunlight by a volcanic eruption on the Fuji island

d) Ripening of fruits due to the hot local winds in winter seasons

Answer: b

Explanation:

• When two storms move towards another, an uncommon phenomenon called the Fujiwhara
Effect can happen. This phenomenon is also known as the Fujiwhara effect, Fujiwhara interaction, or
binary interaction. The effect was first described in 1921 by Sakuhei Fujiwhara, a Japanese
meteorologist.

• When two hurricanes spinning in the same direction pass close enough to each other, they
begin an intense dance around their common centre. If one hurricane is a lot stronger than the
other, the smaller one will orbit it and eventually come crashing into its vortex to be absorbed. Two
storms closer in strength can gravitate towards each other until they reach a common point and
merge, or merely spin each other around for a while before shooting off on their paths. On rare
occasions, the effect is additive when the hurricanes come together, resulting in one larger storm
instead of two smaller ones.

Therefore, option (b) is the correct answer.

Relevance: Recently Fujiwhara effect was noticed in the two tropical cyclones

Subject: Geography| Natural Hazards and Disasters

Tags: Factual, Conceptual

Q97. With reference to Polar Vortex, consider the following statements:

1. It expands due to the weakening of the jet stream and brings extreme cold waves to North
America.

2. All the cold weather in the Northern hemisphere is due to its permanent nature in the Arctic.

3. It becomes stable and remains confined within the jet stream influence during the winter.

Which of the statements given above is/are correct?

a) 1 and 2 only

b) 1 only

c) 2 and 3 only

d) 3 only

Answer: b

Explanation:
• The polar vortex is a large area of low pressure and cold air surrounding both of the Earth's
poles. It always exists near the poles, but weakens in summer and strengthens in winter. The term
"vortex" refers to the counter-clockwise flow of air that helps keep the colder air near the Poles.

• When the low-pressure system is strong and healthy, it keeps the jet stream travelling
around Earth in a circular path. The jet stream is a band of reliably strong wind that plays a key role
in keeping colder air north and warmer air south. But when the vortex weakens, part of the
weakened low-pressure system can break off. This breaking-off process is what can cause colder
temperatures in North America. So, statement 1 is correct.

• Cold temperatures due to the polar vortex are not rare in the Northern region near the
arctic. But not all cold weather comes from the polar vortex. Although cold air from the polar vortex
can be pushed south, it typically remains parked in polar regions. It takes pretty unusual conditions
for the vortex to weaken or migrate far south. Other weather conditions can cause cold arctic
weather to travel south, too. So, statement 2 is not correct.

• In winter, the polar vortex sometimes becomes less stable and expands. Many times during
winter in the northern hemisphere, the north polar vortex will expand, sending cold air southward
with the jet stream. This is called a polar vortex event, defined as the “breaking off” of a part of the
vortex. So, statement 3 is not correct.

Therefore, option (b) is the correct answer.

Relevance: Meteorologists have blamed polar vortex for the bitter cold that has descended on much
of the central and eastern United States

Subject: Geography| Weather and Climate and Atmospheric Circulation

Tags: Conceptual

Q98. Which the following statements is/are correct with reference to the ocean currents?

1. The Kuroshio Current is a cold current running from Taiwan to the Bering Strait.

2. The Benguela current is a warm current in the Pacific Ocean.

3. El Nino current makes the Peruvian coastal desert green and an ideal harvest of cotton.

Select the correct answer using the code given below.

a) 1 and 2 only

b) 1 only

c) 2 and 3 only

d) 3 only

Answer: d

Explanation:

• Kuroshio system is a warm current composed of several currents and drifts similar to the
Gulf stream system of the Atlantic ocean. This system runs from Taiwan to the Bering Strait and
consists of the Kuroshio current, the Kuroshio extension, the North Pacific drift, the Tsushima
current and the counter Kuroshio current. So, statement 1 is not correct.
• Winds blowing over the cold current discourage rainfall. This is seen in the Kalahari Desert
along the western coast of South Africa in the Atlantic ocean and the Atacama desert along the
western coast of South America in the Pacific ocean because of the existence of Benguela and Peru
currents respectively. So, statement 2 is not correct.

• A subsurface warm current known as El Nino current flows from north to south between 3°S
and 36°S latitudes at a distance of about 180km from the Peruvian coast. The southward shifting of
the counter equatorial warm current during southern winter gives birth to the El Nino current. The
temperature on the Peruvian coast does not fall considerably because of this current. The heavy
rainfall associated with the strong El Nino event makes coastal Peruvian deserts green and there is a
rich harvest of cotton, coconuts and bananas but there is an oceanic biological disaster. So,
statement 3 is correct.

Therefore, option (d) is the correct answer.

Subject: Geography|Oceanography

Tags: Conceptual

Q99. With reference to freshwater ecology, which of the following are Lentic Ecosystems?

1. Swamps

2. Rivers

3. Bogs

4. Springs

5. Ponds

Select the correct answer using the code given below.

a) 1, 2 and 4 only

b) 2 and 4 only

c) 1, 3 and 5 only

d) 1, 2, 3, 4 and 5

Answer: c

Explanation:

• The freshwater ecology refers to the relationships that living organisms have with each other
in their instinctive environment, thus forming a well-balanced ecosystem. Though they occupy a
relatively small portion of the earth’s surface as compared to other marine and terrestrial habitats,
they are of much importance to a variety of life forms on the earth. Freshwater ecology, based on
habitat condition, is categorised into two types: (i) Standing water or lentic: for example ponds,
swamps and bogs etc., and (ii) Flowing water or lotic: classical examples being rivers, streams,
springs etc.

Therefore, option (c) is the correct answer.

Subject: Geography| the hydrosphere


Tags: Conceptual

Q100. Consider the following statements:

1. Uttar Pradesh shares its border with the maximum number of states.

2. Sikkim shares its border with one state only.

3. Assam shares its border with six states only.

Which of the statements given above is/are correct?

a) 1 only

b) 1 and 2 only

c) 2 and 3 only

d) 1, 2 and 3

Answer: b

Explanation:

• Uttar Pradesh is the Indian State which shares its borders with 8 states and 1 union territory,
which is maximum. Delhi, Haryana, Rajasthan, Madhya Pradesh, Jharkhand, Bihar, Uttarakhand,
Himachal Pradesh and Chhattisgarh. So, statement 1 is correct.

• Sikkim and Meghalaya are the only two states having borders with only one state – Sikkim
has borders with West Bengal only, while sharing its international borders with three nations –
China, Bhutan and Nepal. So, statement 2 is correct.

• Assam shares Inter-State boundaries with 7 states - Arunachal Pradesh, Nagaland, Manipur,
Mizoram, Tripura, Meghalaya and West Bengal. And International border with Bangladesh. So,
statement 3 is not correct.

Therefore, option (b) is the correct answer.

Subject: Geography | Map

Tags: Factual

You might also like